Você está na página 1de 274

VERBAL ABILITY

SESSION - 1
SENTENCE CORRECTION - I
Sentence Correction

The 4 Steps for Sentence Correction


1. Take a First Glance
2. Read the Sentence
3. Find a Starting Point
4. Eliminate Answers

© 2016 SMART Training Resources Pvt. Ltd.


 Identify what SC concept is being tested.
For example, if the concept tested is parallelism, try to figure out what
items should be in parallel. If it is modifiers, identify the referent of
each clause and their correct placement.
 Treat options A, B, C, D, E equally.
Even if you feel sure that a particular answer choice is right, don’t make
up your mind till you have really looked at the other options.
 Look for subject verb mismatches.
Subject Verb Agreement is one of the easiest errors to identify in any SC
questions, at least 1 option can be eliminated in this way. So look for
these first.

© 2016 SMART Training Resources Pvt. Ltd.


Either my neighbour or her children is coming for dinner. ans: are
Neither they nor I were mistaken. ans: was

Errors in use of Pronouns:


e.g.

One must not reveal his secrets to all. ans: one’s secrets to all

- ‘one’ as a subject should use one’s.

© 2016 SMART Training Resources Pvt. Ltd.


 Don’t get confused by pronoun ambiguity.
Ambiguous pronouns are a problem only if the meaning of the sentence
is affected because of the pronoun. Use this concept only if all else fails.
Example 1
The common cold is one of our most indiscriminate diseases;
A
it makes no distinction between you and me, millionaires and paupers,
B C D
or athletes and couch potatoes. No error
E

© 2016 SMART Training Resources Pvt. Ltd.


Step 1: Read the whole sentence
• Reading this sentence, I don't notice anything jump out, although D
sounds kind of weird.
• Even if that answer seems tempting we can't pick it until we
determine what the error is.

© 2016 SMART Training Resources Pvt. Ltd.


Step 2: Check each answer choice
• First let's check B for errors, since it includes both a pronoun and
verb. Make sure to confirm that the verb works. Let's go through the
possible errors for choice B one at a time.

• Is the pronoun in the correct case? Yes. "It" is acting as subject


for the verb "makes," which is perfectly acceptable.

• Does the pronoun agree with its antecedent? Yes. "It" is


standing in for "the common cold," which is a singular non-
gendered noun.

© 2016 SMART Training Resources Pvt. Ltd.


• Is the verb correctly conjugated? Yes. "It" is a singular pronoun
and "makes" is a singular verb.

• Is the verb in the correct tense? Yes. It's in the present tense,
which is consistent with the rest of the sentence .

• Let's move on to choice D, "you and me," which also includes


pronouns. This section sounds a little weird, but remember that
pronoun order doesn't matter—we only need to check agreement
and case, like we did above.

© 2016 SMART Training Resources Pvt. Ltd.


• Are the pronouns in the correct case? This question is a bit
trickier. It would sound just fine to replace "me" with "I"—"it makes
no distinction between you and I"—but "between" is a preposition,
so the object case "me" is correct.

• Although D sounds odd, it doesn't contain an error. We can


eliminate it and move onto C, which includes a noun and
preposition. We'll check the preposition first since it's more likely to
include an error.

© 2016 SMART Training Resources Pvt. Ltd.


• Is the preposition idiomatically correct? Yes. "Distinction
between" is a common expression.

• Does it incorrectly complete a word pair? No. "Between" is


correctly followed by two prepositions connected by "and."

• Is the sentence consistent in its use of plural and singular


nouns? Yes. There's no issue with agreement.

• Since we didn't find any issues with C, we'll move onto checking the
last answer, A, "one of our most."

© 2016 SMART Training Resources Pvt. Ltd.


• Is the phrase idiomatically correct? Yes. "One of our most
indiscriminate diseases" is a clear and acceptable phrase.

• Is the superlative/comparative correct? Yes. The common


cold is being compared to all of the other diseases, so "most" is the
correct word choice.
• We can conclude that there is no error in A, so we've ruled out all the
choices, leaving only E, "no error."

• Step 3: Confirm what the error in your answer is


• This sentence doesn't have an error, so the correct answer is E.

© 2016 SMART Training Resources Pvt. Ltd.


Example 2
The architect’s research shows that even when builders construct houses
A
of stone, they still use the hammer more than any tool. No error
B C D E
Step 1: Read the whole sentence
Step 2: Check each answer choice
We'll start with A,
Is the verb correctly conjugated?
Is the verb in the correct tense
Is the preposition idiomatically correct?
Does it incorrectly complete a word pair?
Is the noun part of a faulty comparison?
Is the sentence consistent in its use of plural and singular nouns?

© 2016 SMART Training Resources Pvt. Ltd.


Is the modifier of the correct type? "Still" is describing when the
builders use the hammer, so it's correct as an adverb.

Are the comparative and superlatives [-er and –est] used


appropriately?

Is the noun part of a faulty comparison?


That comparison seem fine, but it actually includes an error. A hammer
is a tool, so it must be compared with any other tool, rather than just
"any tool." D is the answer.

© 2016 SMART Training Resources Pvt. Ltd.


Step 3: Confirm what the error in your answer is
• Choice D is the answer because it creates an illogical comparison.
We've also definitely ruled out the other choices (except no error).

© 2016 SMART Training Resources Pvt. Ltd.


QUESTION TYPES:
I. SPOTTING ERRORS
II. SENTENCE CORRECTION
III. IDENTIFYING CORRECT AND INCORRECT USAGE

© 2016 SMART Training Resources Pvt. Ltd.


I. SUBJECT VERB AGREEMENT
Directions for Q1 to Q3: Identify the grammatically correct
sentence(s) in the question.
1. A. The magistrate has issued a summon asking me to attend his court
on Monday.
B. My friend gives me many advices regarding my career.
C. Our HOD has purchased new furniture for our department.
D. He is one of the notorious troubles-makers in the town.
(a) A, C (b) A, B
(c) C (d) A, C, D
Directions for Q1 to Q3: Identify the grammatically correct
sentence(s) in the question.
2. A. The H.M.T. factory has the latest machinery.
B. The physician has given me much information about the disease.
C. My hairs have turned grey recently.
D. Four thiefs broke into my cousin brother’s house yesterday night.
(a) A, C (b) A, B
(c) B (d) A, C, D
Directions for Q1 to Q3: Identify the grammatically correct
sentence(s) in the question.
3.A. This scissor will not cut such a hard wire.
B. His little daughter is capable of many a mischief.
C. I have seen many cannons in the military parade ground.
D. The beggar is begging for alm.
(a) B, C (b) A, D
(c) A, C (d) B, D
Directions for Q4 to Q7: Identify the grammatically incorrect
sentence(s) in the question.
4. A. The first innings is over just now.
B. The company wants to dispose of the equipment.
C. She took great pain to write the essay properly.
D. Rakesh, the banker’s son, is very intelligent.
(a) A, B, D (b) B, C
(c) A, B (d) C
Directions for Q4 to Q7: Identify the grammatically incorrect
sentence(s) in the question.
5. A. These sceneries of Kashmir are beautiful.
B. These golds have been brought from South Africa.
C. My brothers-in law will come to our house next week.
D. He lost all his luggages on his way home.
(a) C (b) A, B, D
(c) A, C (d) C, D
Directions for Q4 to Q7: Identify the grammatically incorrect
sentence(s) in the question.
6.A. For many students, examinations are a major source of stress.
B. It can be mentally incapacitating and can drastically reduce one’s
efficiency.
C. Systematic approach right from the beginning of the academic year
boost self confidence.
D. It helps one to perform well at the examination.
(a) Only A (b) A and B
(c) B and C (d) C and D
Directions for Q4 to Q7: Identify the grammatically incorrect
sentence(s) in the question.
7. A. The true role of management is to make risk-taking possible.
B. Executives can’t order their staff to be creative.
C. They have to provide the conditions where creativity flourish.
D. Such conditions includes strong staff morale and the conviction that
good work will be rewarded.
(a) Only D (b) C and D
(c) B and D (d) A and B
Directions for Q8 to Q13: Replace the underlined part of the sentence
with a grammatically correct part.
8. Out of America’s fascination with all things antique have grown a market
for by gone styles of furniture and fixtures that are bringing back the chaise
lounge, the overstuffed sofa and the claw-footed bathtub.
(a) things antique have grown a market for bygone styles of furniture and
fixtures that are bringing
(b) things antique has grown a market for bygone styles of furniture and
fixtures that is bringing
(c) things that are antiques has grown a market for by gone styles of
furniture and fixtures that bring
(d) antique things have grown the market for bygone styles of furniture and
fixtures that are bringing
9. Diabetes, together with its serious complications, ranks as the
nation’s third leading cause of death, surpassed only by heart disease
and cancer.
(a) ranks as the nation’s third leading cause of death, surpassed only
(b) rank as the nation’s third leading cause of death, only surpassed
(c) has the rank of the nation’s third leading cause of death, only
surpassed
(d) are the nation’s third leading causes of death, surpassed only
10. Efforts to equalize the funds available to school districts, a major
goal of education reformers in many states in the 1970’s, has not
significantly reduced the gaps existing between the richest and poorest
districts.
(a) has not significantly reduced the gaps existing
(b) has not been significant in reducing the gap that exists
(c) has not made a significant reduction in the gap that exists
(d) have not significantly reduced the gap that exists
11. While larger banks can afford to maintain their own data-processing
operations, many smaller regional and community banks are finding
that the cost associated with upgrading data-processing equipment and
with the development and maintenance of new products and technical
staff are prohibitive.
(a) cost associated with
(b) costs associated with
(c) costs arising from
(d) cost of
Please, miss, ‘trousers’ should surely be singular
at the top and plural at the bottom!
12. While all states face similar industrial waste problems, the
predominating industries and the regulatory environment of the states
obviously determines the types and amounts of waste produced, as well as
the cost of disposal.
(a) all states face similar industrial waste problems, the predominant
industries and the regulatory environment of each state obviously
determine
(b) each state faces a similar industrial waste problem, their predominant
industries and regulatory environment obviously determine
(c) all states face a similar industrial waste problem; their predominating
industries and regulatory environment obviously determines
(d) each state faces similar industrial waste problems, the predominant
industries and the regulatory environment of each state obviously
determines
13. The peaks of a mountain range, acting like rocks in a stream bed,
produce ripples in the air flowing over them; the resulting flow pattern,
with crests and troughs that remain stationary although the air that
forms them is moving rapidly, are known as “standing waves.”
(a) crests and troughs that remain stationary although the air that
forms them is moving rapidly, are
(b) stationary crests and troughs although they are formed by rapidly
moving air, is
(c) crests and troughs that remain stationary although the air that
forms them is moving rapidly, is
(d) stationary crests and troughs although the air that forms them is
moving rapidly, are
Directions Q14 and Q15: Choose the sentence which is structurally correct.
14. A. Most of the applications re used for security systems in automobiles and
are increasingly in use for anti-skid braking, roll over systems, anti-theft systems,
electronic car braking systems, and the like.
B. Most of the applications are used for security systems in automobiles and
is increasingly in use for anti- skit braking, roll over systems, anti-theft systems,
electronic car braking systems, like so many others.
C. Most of the applications are used for security systems in automobiles but
are increasingly being used for anti-skid braking, roll over systems, anti-skid
braking, roll over systems, anti- theft systems, electronic car braking systems, and
like others.
D. Most of the applications are used for security systems in automobiles but
is increasingly being used for anti-skid braking, roll over systems, anti-theft
systems, electronic car braking systems and so many others.
(a) A (b) B (c) C (d) D
Directions Q14 and Q15: Choose the sentence which is structurally correct.
15. A. Politics in the Pacific island nation of Fiji have come a long way, with a
Supreme Court ruling helping to heal the wounds inflicted by three coups since Fiji
won independence from Britain in 1970.
B. Politics in the Pacific island nation of Fiji has come a long way, with a
Supreme Court ruling helping to heal the wounds inflicted by three coups since Fiji
won independence from Britain in 1970.
C. Politics in the Pacific island nation of Fiji have come a long way, with a
Supreme Court ruling helping to heal the wounds inflicted with three coups since
Fiji won independence from Britain in 1970.
D. Politics in the Pacific island nation of Fiji has come a long way with a
Supreme Court ruling helping to heal the wounds inflicted by three coups since Fiji
won independence upon Britain in 1970.
(a) A (b) B (c) C (d) D
Directions for Q16 to Q25: Identify the part of the sentence which is
grammatically incorrect.

16. Magic realism is one (a) / of the latest (b) / addition to good
literature (c) / published in recent times. (d) / No error (e)

17. Neither Prannoy nor his wife (a) / were aware (b) / of the
arrangements made (c) / for their journey. (d) / No error (e)
Directions for Q16 to Q25: Identify the part of the sentence which is
grammatically incorrect.

18. In these days of inflation (a) / a ten rupee’s note (b) / will not buy
you (c) / even an ordinary meal. (d) / No error (e)

19. I am not (a) / one of those (b) / who believe (c) / everything I hear.
(d) / No error (e)
Directions for Q16 to Q25: Identify the part of the sentence which is
grammatically incorrect.

20. All one can gather (a) / from the children (b) / are that there were
(c) / a loud noise and smoke. (d) / No error (e)

21. The number of people (a) / applying were so large (b) / that the
college had to (c) / stop issuing application forms. (d) / No error (e)
Directions for Q16 to Q25: Identify the part of the sentence which is
grammatically incorrect.

22. The secretary and treasurer (a) / were (b) / not (c) / present at
today’s meeting. (d) / No error (e)

23. No less (a) / than 20 persons (b) / were killed (c) / in the air crash.
(d) / No error (e)
Directions for Q16 to Q25: Identify the part of the sentence which is
grammatically incorrect.

24. These kinds (a) / of bags seems to be (b) / expensive but it is relatively
economical (c) / to maintain them. (d) / No error (e)

25. Many a man (a) / have realized (b) /that real happiness lies in (c) /
making sacrifices and not in personal aggrandizement. (d) / No error (e)
End of Session - 1
Thank You

© 2016 SMART Training Resources Pvt. Ltd.


VERBAL ABILITY
SESSION - 2
SENTENCE CORRECTION - II
Directions for Q26 to Q30: Which is the preferable form in each
sentence below?

26. Everyone, but the artists, is/are in the hall.

27. The results of this complicated lawsuit remain/remains to be


seen.

28. I don’t suppose that either of them care/cares one way or another.
Directions for Q26 to Q30: Which is the preferable form in each
sentence below?

29. That is one of those setbacks that really is/are unavoidable.

30. The chairman, together with his two young daughters, have/has
accepted our invitation.
II. TENSES
Directions for Q31 to Q34: Complete the following sentences with
the correct tense forms.
31. We travelled Northwards to Goa, where we _________ our two
days in a chauffeur-driven car looking for a hotel.
(a) spend
(b) will spend
(c) spent
(d) spending
Directions for Q31 to Q34: Complete the following sentences with
the correct tense forms.

32. I have chosen to live with Yadav because I _________


(a) had loved him
(b) will be loving him
(c) loved him
(d) love him
Directions for Q31 to Q34: Complete the following sentences with
the correct tense forms.
33. When Mrs. Modi came to the organization in 2005, Mr. Naik
_________ there for two years.
(a) has been working
(b) will be working
(c) had already been working
(d) is working
Directions for Q31 to Q34: Complete the following sentences with
the correct tense forms.

34. They ____________ their lunch by the time you get there.
(a) have eaten
(b) will have eaten
(c) are eating
(d) had eaten
Directions for Q35: Choose the grammatically correct sentence.
35. A. India can be the largest knowledge and skill-based service provider
to the world within the next two decades, just as China seemed to have
become the largest of low-cost manufacturers.
B. India can be the largest knowledge and skill-based service provider
to the world within the next two decades, just as China seems to have
become the largest of low-cost manufacturers.
C. India can be the largest knowledge and skill-based service
providers to the world within the next two decades, just as China seemed to
have become the largest of low-cost manufacturers.
D. India could be the largest knowledge and skill-based service
provider to the world within the next two decades, just as China seemed to
have become the largest of low-cost manufacturers.
(a) A (b) B (c) C (d) D
Directions for Q36 to Q45: Replace the underlined part of the
sentence with a grammatically correct option.
36. If the magazines will be published last week, why haven’t they
been bought yet?
(a) was published
(b) were published
(c) would have been published
(d) are being published
Directions for Q36 to Q45: Replace the underlined part of the
sentence with a grammatically correct option.

37. Does he thought about the consequences if he refuses the offer


of someone so powerful?
(a) Does he thought about
(b) Does he know
(c) Will he have to face
(d) Had he thought about
38. Despite exile, imprisonment and the suppression of almost
every one of his books by the minions of Church and State, Voltaire
forged fiercely a path for his truth, until at last kings, Popes and
Emperors catered to him, thrones trembled before him and half the
world would be listening to catch his every word.
(a) half the world would be listening to catch his every word
(b) half the world listened to catch his every word
(c) half the world have been listening to catch his every word
(d) half the world had listened to catch his every word
39. Of all the vast tides of migration that have swept through
history, may be none is more concentrated as the wave that brought 12
million immigrants onto American shores in little more than three
decades.
(a) perhaps none was more concentrated than
(b) maybe none is more concentrated as
(c) it may be that none is more concentrated as
(d) may be it is none that was more concentrated than
40. Rising inventories, when unaccompanied correspondingly by
increases in sales, can lead to production cutbacks that would hamper
economic growth.
(a) if not accompanied by corresponding increases in sales, can lead
(b) when unaccompanied correspondingly by increases in sales, can
lead
(c) when not accompanied by corresponding increases in sales, possibly
leads
(d) when they were unaccompanied by corresponding sales increases,
can lead
41. The Iroquois were primarily planters, but supplementing their
cultivation of maize, squash, and beans with fishing and hunting.
(a) but supplementing
(b) and had supplemented
(c) and eventhough they supplemented
(d) although they supplemented
42. Scientists have recently discovered what could be the largest
and oldest living organism on Earth, a giant fungus that is an
interwoven filigree of mushrooms and root like tentacles spawned by a
single fertilized spore some 10,000 years ago and extending for more
than 30 acres in the soil of a Michigan forest.
(a) extending
(b) extends
(c) extended
(d) it extended
43. Five fledgling sea eagles left their nests in Western Scotland
this summer, bringing to 34 the number of wild birds successfully
raised since transplants from Norway began in 1975.
(a) bringing
(b) and brings
(c) and it brings
(d) and it brought
44. Affording strategic proximity to the Strait of Gibraltar, Morocco
was also of interest to the French throughout the first half of the
twentieth century because they assumed that if they did not hold it, their
grip on Algeria was always insecure.
(a) if they did not hold it, their grip on Algeria was always insecure
(b) without it their grip on Algeria would never be secure
(c) their grip on Algeria was not ever secure if they did not hold it
(d) without that, they could never be secure about their grip on Algeria
45. His studies of ice-polished rocks in his Alpine homeland, far
outside the range of present-day glaciers, led Louis Agassiz in 1837 to
propose the concept of an age in which great ice sheets had existed in
now currently temperate areas.
(a) in which great ice sheets had existed in now currently temperate
areas
(b) in which great ice sheets existed in what are now temperate areas
(c) when great ice sheets existed where there were areas now temperate
(d) when great ice sheets had existed in current temperate areas
Directions for Q46 to Q50: Identify the part of the sentence which
is grammatically incorrect.
46. No country can long endure (a) / if its foundations (b) / were not
laid deep (c) / in the material prosperity. (d)

47. While Mahendra was away (a) / on a long official tour (b) / his
office receive an important letter (c)/ which was marked ‘Urgent’. (d)

48. Looking forward (a)/ to meeting (b) / you (c) / soon. (d)
Directions for Q46 to Q50: Identify the part of the sentence which
is grammatically incorrect.
49. Thinking that he has finally found (a) / someone with similar
interests, (b) / the scholar tried to strike up a conversation. (c) / No
error (d)

50. Earlier this year, (a) / Constantan had entered the news via a
video-taped interview (b) / telecasted by commercial television
channel. (c) / No error (d)
End of Session - 2
Thank You

© 2016 SMART Training Resources Pvt. Ltd.


VERBAL ABILITY
SESSION - 3
SENTENCE CORRECTION - III
III. PRONOUN
Directions for Q51 and Q52: Identify the grammatically correct
sentence(s).
51. A. Between you and me, the secretary is not a gentleman.
B. One should do your duty honestly and sincerely.
C. Good students like you and he should study regularly.
D. You, he and I are in the wrong.
(a) A, C (b) B, D
(c) B, C (d) A
Directions for Q51 and Q52: Identify the grammatically correct
sentence(s).
52. A. Only you and him can do this work fast.
B. If I were him, I would not misbehave like this.
C. Everyone announced one’s plans in the presence of the
President.
D. She helped everyone of those boys in doing his work.
(a) A, B (b) B, C
(c) D (d) C, D
Directions for Q53 to Q55: Identify the grammatically incorrect
sentence(s).
53. A. Every teacher and every student should do his duty.
B. Neither of the boys have submitted their records.
C. Them availed of the opportunity.
D. Those two companies always help one another.
(a) A, B, C (b) B, C, D
(c) C, D (d) B, D
Directions for Q53 to Q55: Identify the grammatically incorrect
sentence(s).
54. A. India and Pakistan should cooperate with each other in this
matter.
B. Dear students, enjoy yourself the holidays.
C. The committee were divided in its opinion regarding this issue.
D. The team, after taking its bath, has gone for practice of cricket.
(a) B, C (b) C, A
(c) A, D (d) A, B, C
Directions for Q53 to Q55: Identify the grammatically incorrect
sentence(s).
55. A. My opinion is the same as your.
B. We were told to let only you and she enter
C. This is the scientist which won the first prize
D. His problems are the same as mine.
(a) B, C, D (b) A, B, C
(c) C, D, A (d) All the above
Directions for Q56 to Q60: Identify the part of the sentence that
has an error in it.
56. (a) Well, I spent six or seven years / (b) after high school / (c)
trying to find a job for me / (d) but could not succeed in it. / (e) No
error

57. (a) The purpose of this book, / (b) however, is not to discuss /
(c) these basic issues / (d) in its various aspects. / (e) No error

58. (a) Every man and woman / (b) should vote / (c) for the
candidate / (d) of their choice. / (e) No error
Directions for Q56 to Q60: Identify the part of the sentence that
has an error in it.

59. (a) Cannot one do / (b) what one / (c) likes to do / (d) on his
own? / (e) No error

60. (a) There’s Mr. Shanu / (b) whom they say / (c) is the best
singer / (d) in the country. / (e) No error
IV. CONJUNCTIONS
Directions for Q61 to Q64: Identify the grammatically incorrect
sentence(s).
61.
1. Although she is foolish but people like her.
2. The accident took place as the train was crossing a bridge.
3. Moving on the grass I saw a snake.
4. However fast he may run, he cannot catch the train.
(a) 1, 2, 3 (b) 2, 3
(c) 4, 2, 1 (d) 1, 2
Directions for Q61 to Q64: Identify the grammatically incorrect
sentence(s).
62.
1. Work hard lest you may not fail.
2. Scarcely he had entered the room, I recognized him.
3. Supposing if he misses the train, will he come back?
4. Unless you do not pay attention to what I say, you will not succeed.
(a) 1, 2 (b) 2, 3
(c) 1, 3, 4 (d) 1, 2, 3, 4
Directions for Q61 to Q64: Identify the grammatically incorrect
sentence(s).
63.
1. The teacher asked John that why he was late.
2. It is not certain that he will come.
3. No other country but India has spoken against the racist regime of
South Africa.
4. He behaves as if he is mad.
(a) 1, 2, 3 (b) 1, 3, 4
(c) 1, 2 (d) 3, 4
Directions for Q61 to Q64: Identify the grammatically incorrect
sentence(s).
64. 1. I have neither seen him nor his brother.
2. Lions are both found in Asia and Africa.
3. Do you know where he lives?
4. He did not speak loudly and clearly.
(a) 1, 2 (b) 1, 2, 4
(c) 2, 3, 4 (d) 1, 2, 3
Directions for Q65 and Q66: Identify the part of the sentence
which is grammatically incorrect.
65. (A) Although they listen to me / (B) but their actions / (C)
prove otherwise. / (D) No error

66. (A) No sooner did the sun rise / (B) when we took a hasty
breakfast / (C) and resumed the journey. / (D) No error
V. PREPOSITIONS
Directions for Q67 to Q71: Choose the correct alternative to
complete the sentence.
67. Divide this food (between/among) the five beggars.

68. He worked (to/till) 8 O’clock.

69. He is very popular (among/amongst) us.


V. PREPOSITIONS
Directions for Q67 to Q71: Choose the correct alternative to
complete the sentence.

70. The fort stands (beside/besides) the Yamuna.

71. He will stay with me (for/during) June.


Directions for Q72 to Q76: Identify the part of the sentence which is
grammatically incorrect.

72. (A) He is in the habit of going out / (B) on a morning / (C) walk
regularly. / (D) No error

73. (A) The management did not select him / (B) as he was / (C)
not eligible to the post. / (D) No error
Directions for Q72 to Q76: Identify the part of the sentence which is
grammatically incorrect.

74. (A) Both the form and substance of what he says / (B) about his life
and work / (C) testify for a great reserve of inner energy. / (D) No error

75. (A) Young children should not sit / (B) close to the TV set / (C) as it
affects on their eyes. / (D) No error
Directions for Q72 to Q76: Identify the part of the sentence which is
grammatically incorrect.

76. (A) When the teachers are on strike / (B) and a notice to this effect
is pasted on the college gate / (C) there is no sense to go there. / (D) No
error
End of Session - 3
Thank You

© 2016 SMART Training Resources Pvt. Ltd.


VERBAL ABILITY
SESSION - 4
SENTENCE CORRECTION - IV
VI. PARALLELISM
Directions for Q77 to Q86: Replace the underlined part of the question with a
grammatically correct option.
77. The result of two recent unrelated studies support the idea that dolphins may
share certain cognitive abilities with humans and great apes; the studies indicate
dolphins as capable of recognizing themselves in mirrors an ability that is often
considered a sign of self-awareness and to grasp spontaneously the mood or intention of
humans.
(a) dolphins as capable of recognizing themselves in mirrors – an ability that is
often considered a sign of self-awareness – and to grasp spontaneously
(b) dolphins’ ability to recognize themselves in mirrors – an ability that is often
considered as a sign of self-awareness and of spontaneously grasping
(c) that dolphins have the ability of recognizing themselves in mirrors an ability
that is often considered as a sign of self-awareness and spontaneously grasping
(d) that dolphins are capable of recognizing themselves in mirrors – an ability that
is often considered a sign of self-awareness – and of spontaneously grasping
78. Federal authorities involved in the investigation have found the
local witnesses are difficult to locate, reticent, and are suspicious of
strangers.
(a) the local witnesses are difficult to locate, reticent, and are
(b) local witnesses to be difficult to locate, reticent, and are
(c) that local witnesses are difficult to locate, reticent, and
(d) local witnesses are difficult to locate and reticent, and they are
79. Although early soap operas were first aired on evening radio in the
1920’s, they had moved to the daytime hours of the 1930’s when the
evening schedule became crowded with comedians and variety shows.
(a) were first aired on evening radio in the 1920’s, they had moved to
the daytime hours of the 1930’s
(b) were first aired on evening radio in the 1920’s, they were moved to
the daytime hours in the 1930’s
(c) were aired first on evening radio in the 1920’s, moving to the
daytime hours in the 1930’s
(d) aired on evening radio first in the 1920’s in the daytime hours
80. The only way for growers to salvage frozen citrus is to process them
quickly into juice concentrate before they rot when warmer weather
returns.
(a) to process them quickly into juice concentrate before they rot
when warmer weather returns
(b) if they are quickly processed into juice concentrate before
warmer weather returns to rot them
(c) for them to be processed quickly into juice concentrate before
the fruit rots when warmer weather returns
(d) to have it quickly processed into juice concentrate before
warmer weather returns and rots the fruit.
81. In 1995 Richard Stallman, a well-known critic of the patent
system, testified in Patent Office hearings that, to test the system, a
colleague of his had managed to win a patent for one of kirchhoff’s laws, an
observation about electric current first made in 1845 and now included in
virtually every textbook of elementary physics.
(a) laws, an observation about electric current first made in 1845 and
(b) laws, which was an observation about electric current first made in
1845 and it is
(c) laws, namely, it was an observation about electric current first
made in 1845 and
(d) laws, an observation about electric current first made in 1845, it is
82. Critics contend that the new missile is a weapon whose
importance is largely symbolic, more a tool for manipulating people’s
perceptions than to fulfill a real military need.
(a) for manipulating people’s perceptions than to fulfill
(b) for manipulating people’s perceptions than for fulfilling
(c) to manipulate people’s perceptions rather than that it fulfilling
(d) to manipulate people’s perceptions rather than fulfilling
83. It will not be possible to implicate melting sea ice in the coastal
flooding that many global warming models have projected: just like a
glass of water that will not overflow due to melting ice cubes, so melting
sea ice does not increase oceanic volume.
(a) like a glass of water that will not overflow due to melting ice
cubes
(b) like melting ice cubes that do not cause a glass of water to
overflow`
(c) a glass of water will not overflow because of melting ice cubes
(d) as melting ice cubes do not cause a glass of water to overflow
84. By developing the Secure Digital Music Initiative, the recording industry
associations of North America, Japan, and Europe hope to create a standardized
way of distributing songs and full-length recordings on the Internet that will protect
copyright holders and foil the many audio pirates who copy and distribute digital
music illegally.
(a) of distributing songs and full - length recordings on the Internet that will
protect copyright holders and foil the many audio pirates who copy and distribute
(b) of distributing songs and full-length recordings on the Internet and to
protect copyright holders and foiling the many audio pirates copying and
distributing
(c) for distributing songs and full - length recordings on the Internet while it
protects copyright holders and foils the many audio pirates who copy and distribute
(d) to distribute songs and full - length recordings on the Internet while they
will protect copyright holders and foil the many audio pirates copying and
distributing
85. The end of the eighteenth century saw the emergence of prize-
stock breeding, with bulls and cows receiving awards, fetching
unprecedented prices, and excited enormous interest whenever they
were put on show.
(a) it excited
(b) exciting
(c) would excite
(d) it had excited
86. The report recommended that the hospital should eliminate
unneeded beds, expensive services should be consolidated, and use
space in other hospitals.
(a) should eliminate unneeded beds, expensive services should be
consolidated, and use space in other hospitals
(b) should eliminate unneeded beds, expensive services should be
consolidated, and other hospitals’ space be used
(c) eliminate unneeded beds, consolidate expensive services, and
use other hospitals’ space
(d) eliminate unneeded beds, consolidated, and to use space in
other hospitals
VII. MODIFIERS
Directions for Q87 to Q89: Identify the grammatical flaws in the
following sentences and correct them.
87. Smoking a big cigar, the baby was admired by its father.

88. If the baby does not thrive on fresh milk, it should be boiled.

89. At the age of twenty, my father let me drive his car.


Directions for Q90: Spot out the grammatically correct sentence.
90.
(a) Manufactured in Japan, Asmi was delighted with the fine
quality of the leather bags.
(b) The patient was referred to a psychiatrist with several
emotional problems.
(c) While climbing the tree, the bottle dropped.
(d) Do not sit on the chair unless it is fully assembled.
VIII. IDIOMS AND PHRASES
Directions for Q91 to Q92: In each of the sentences, a part of the sentence is left
blank, beneath each sentence; four different ways of completing the sentence are
indicated. Choose the idiom for each blank that best fits the meaning of the sentence
as a whole.
91. Manoj was considered to be a _________ at successfully organizing
various cultural and sports events when he was in the senior college.
(a) bundle of nerves (b) bull in a China shop
(c) dab hand (d) chip off the old block

92. The victory stretch of the Australians in the World Cup was _________
for the Indians who were runners up.
(a) a cross to bear (b) a double whammy
(c) a finger in the pie (d) one in the eye
IX. ARTICLES
Directions for Q93 to Q100: Put a/an, the or zero article (-) in the
spaces.
93. A special award was given to …………… novelist Ian McMurphy.
(a) a (b) an
(c) the (d) no article

94. I’ve been offered the position of …………… Director of


Personnel.
(a) a (b) an
(c) the (d) no article
Directions for Q93 to Q100: Put a/an, the or zero article (-) in the
spaces.

95. I’ve always wanted to meet ……… Michael Owen.


(a) a (b) an
(c) the (d) no article

96. ‘What make is your computer?’ It’s ………… Mac.’


(a) a (b) an
(c) the (d) no article
Directions for Q93 to Q100: Put a/an, the or zero article (-) in the
spaces.

97. I’m ………… marketing adviser at Unifleet.


(a) a (b) an
(c) the (d) no article

98. Let me introduce you to ………… Paula Cox.


(a) a (b) an
(c) the (d) no article
Directions for Q93 to Q100: Put a/an, the or zero article (-) in the
spaces.

99. We met our good friend ………… Jean Wools when we were in
…… Plymouth.
(a) a, an (b) an, the
(c) the, a (d) no article

100. I found myself talking to ……… George Bush! Not …… George


Bush, of course, but someone with the same name.
(a) a, the (b) an, the
(c) the, a (d) no article, the
End of Session - 4
Thank You

© 2016 SMART Training Resources Pvt. Ltd.


VERBAL ABILITY
SESSION - 5
SENTENCE CORRECTION - V
Directions for Q101 to Q110: In each sentence, certain words or
phrases have been underlined and labelled (A), (B), (C) or (D). Choose
the underlined word or phrase that is incorrect in each sentence.

101. The (A) Washington Monument is the (B) tribute to the first and
perhaps the greatest (C) president of the (D) United States.

102. Muslims abstain from (A) food and drink during (B) Ramadan, the
(C) ninth month of its (D) calendar.

© 2016 SMART Training Resources Pvt. Ltd.


Directions for Q101 to Q110: In each sentence, certain words or
phrases have been underlined and labelled (A), (B), (C) or (D). Choose
the underlined word or phrase that is incorrect in each sentence.

103. The tomato is (A) an integral part of many (B) regional cuisines,
and it was (C) popular since (D) the 1600’s.

104. The drainage (A) of swamps and other breeding (B) places of (C)
mosquitoes have (D) greatly diminished the incidence of malaria.

© 2016 SMART Training Resources Pvt. Ltd.


Directions for Q101 to Q110: In each sentence, certain words or
phrases have been underlined and labelled (A), (B), (C) or (D). Choose
the underlined word or phrase that is incorrect in each sentence.

105. The shoulder (A) height of the Siberian mammoth, which (B)
roamed throughout (C) the Northern Hemisphere, was about nine foot
(D).

106. In engineering (A), fatigue is the microscopic cracking (B) of


materials, after repeating (C) applications of stress (D).

© 2016 SMART Training Resources Pvt. Ltd.


Directions for Q101 to Q110: In each sentence, certain words or
phrases have been underlined and labelled (A), (B), (C) or (D). Choose
the underlined word or phrase that is incorrect in each sentence.

107. In one of its (A) more useful functions in nature, the common
housefly speeds up (B) the decomposing (C) of dead (D) animals.

108. Magnolia (A) are native (B) to Japan, China, North America and
the Himalayas, and they produce (C) large white, rose or purple flowers
(D).

© 2016 SMART Training Resources Pvt. Ltd.


Directions for Q101 to Q110: In each sentence, certain words or
phrases have been underlined and labelled (A), (B), (C) or (D). Choose
the underlined word or phrase that is incorrect in each sentence.

109. The Pied Piper of Hamelin, the legendary figure of Hamelin,


Germany, rid the town (A) of their (B) rats and mice by leading them
(C) away with his flute playing (D).

110. The pharynx is located on (A) the back of (B) the throat where (C)
the mouth and nose meet (D) the esophagus.

© 2016 SMART Training Resources Pvt. Ltd.


Directions for Q111 to Q120: Each sentence has been divided into
four parts – (A), (B), (C) and (D). Identify which part has an error.

111. After an outbreak of the so-called Mad Cow's disease, (A) / the
United Kingdom was forced (B) / to quarantine its beef exports (C) /
and killing thousands of cows (D).

112. Police departments all over the world (A) / employs computers and
other electronic equipment (B) / to capture the criminals (C) / they
pursue (D).

© 2016 SMART Training Resources Pvt. Ltd.


Directions for Q111 to Q120: Each sentence has been divided into
four parts – (A), (B), (C) and (D). Identify which part has an error.

113. The novels of John Irving (A) / depict common everyday life (B) /
in New Hampshire (C) / where he grown up (D).

114. The Chinese, (A) / who are credited with (B) / the inventing of
gunpowder, (C) / also created the kite and the printing press (D).

© 2016 SMART Training Resources Pvt. Ltd.


Directions for Q111 to Q120: Each sentence has been divided into
four parts – (A), (B), (C) and (D). Identify which part has an error.

115. In 1978, the atoll Enewetak was declared uninhabitable (A) / for at
least 350 years (B) / because of the high levels of radiation (C) /
resulting U.S. nuclear testing (D).

116. Medusa, the more famous (A) / of the three monstrous Gorgons,
(B) / offended Athena (C) who changed Medusa’s hair into snakes (D).

© 2016 SMART Training Resources Pvt. Ltd.


Directions for Q111 to Q120: Each sentence has been divided into
four parts – (A), (B), (C) and (D). Identify which part has an error.

117. With the introduction of the new syllabus, (A) / the number of
colleges reporting (B) / high results are decreasing (C) / year after year
(D).

118. The Maginot Line, (A) / a massive system of fortifications, (B) /


became obsolete before (C) / their completion in 1934 (D).

© 2016 SMART Training Resources Pvt. Ltd.


Directions for Q111 to Q120: Each sentence has been divided into
four parts – (A), (B), (C) and (D). Identify which part has an error.

119. There are lesser buffalos (A) / roaming the Great Plains today (B) /
than during the last century (C) / due to the campaigns of white
Americans against Native Americans (D).

120. The handles of the Big and Little Dippers (A) / extend in opposite
directions, (B) / and when one was upright
(C) / the other is inverted. (D).

© 2016 SMART Training Resources Pvt. Ltd.


Directions for Q121 to Q125: In each sentence below, a part has
been given in bold. Choose the best answer option to replace the bold
part. If no correction or replacement is necessary, choose option (e).
121. The performance of our players was rather worst than I had
expected.
(a) bad as I had expected
(b) worse than I had expected
(c) worse than expectation
(d) worst than was expected
(e) No correction required

© 2016 SMART Training Resources Pvt. Ltd.


Directions for Q121 to Q125: In each sentence below, a part has
been given in bold. Choose the best answer option to replace the bold
part. If no correction or replacement is necessary, choose option (e).

122. Why should the candidates be afraid of English Language is


not clear.
(a) the candidates should be
(b) do the candidates be
(c) should be the candidates
(d) are the candidates
(e) No correction required

© 2016 SMART Training Resources Pvt. Ltd.


Directions for Q121 to Q125: In each sentence below, a part has
been given in bold. Choose the best answer option to replace the bold
part. If no correction or replacement is necessary, choose option (e).

123. Anyone interested in the use of computers can learn much if you
have access to a personal computer.
(a) they have access
(b) access can be available
(c) he or she has access
(d) one of them have access
(e) No correction required

© 2016 SMART Training Resources Pvt. Ltd.


Directions for Q121 to Q125: In each sentence below, a part has
been given in bold. Choose the best answer option to replace the bold
part. If no correction or replacement is necessary, choose option (e).

124. The train will leave at 8.30 p.m. We have been ready by 7.30 p.m.
so that we can reach the station in time.
(a) were
(b) must be
(c) are
(d) should have
(e) No correction required

© 2016 SMART Training Resources Pvt. Ltd.


Directions for Q121 to Q125: In each sentence below, a part has
been given in bold. Choose the best answer option to replace the bold
part. If no correction or replacement is necessary, choose option (e).

125. They examined both the samples very carefully but failed to detect
any difference in them.
(a) some difference in
(b) some difference between
(c) any difference between
(d) any difference among
(e) No correction required

© 2016 SMART Training Resources Pvt. Ltd.


End of Session - 5
Thank You

© 2016 SMART Training Resources Pvt. Ltd.


VERBAL ABILITY
SESSION - 6
SENTENCE CORRECTION - VI
Directions for Q126 to Q130: In each sentence below, a part has
been given in bold. Choose the best answer option to replace the bold
part. If no correction or replacement is necessary, choose option (e).

126. Making friends is more rewarding than to make enemies.


(a) to have enemies
(b) enmity
(c) get enemies
(d) making enemies
(e) No correction required

© 2016 SMART Training Resources Pvt. Ltd.


Directions for Q126 to Q130: In each sentence below, a part has
been given in bold. Choose the best answer option to replace the bold
part. If no correction or replacement is necessary, choose option (e).
127. There’s not so much work to do this week.
(a) many
(b) plenty
(c) less
(d) any
(e) No correction required

© 2016 SMART Training Resources Pvt. Ltd.


Directions for Q126 to Q130: In each sentence below, a part has
been given in bold. Choose the best answer option to replace the bold
part. If no correction or replacement is necessary, choose option (e).

128. Had I realised how close I was to the edge of the road, I would
not have raised the speed of the car.
(a) Had I been realised
(b) If I would have realized
(c) When I realized
(d) Had I had realized
(e) No correction required

© 2016 SMART Training Resources Pvt. Ltd.


Directions for Q126 to Q130: In each sentence below, a part has
been given in bold. Choose the best answer option to replace the bold
part. If no correction or replacement is necessary, choose option (e).
129. They continued to work in the field despite of the heavy rains.
(a) even though there is heavy rain
(b) although heavily rains
(c) in spite the heavy rains
(d) even though it rained heavily
(e) No correction required

© 2016 SMART Training Resources Pvt. Ltd.


Directions for Q126 to Q130: In each sentence below, a part has
been given in bold. Choose the best answer option to replace the bold
part. If no correction or replacement is necessary, choose option (e).

130. Their earnings are such that they find it difficult to make both
ends to meet.
(a) to makings both ends meet
(b) to make both ends for meeting
(c) to make both ends meet
(d) for making both ends to meet
(e) No correction required

© 2016 SMART Training Resources Pvt. Ltd.


Directions for Q131 to Q150: In each sentence below, a part has been
underlined. Beneath the sentence you will find five ways of phrasing the
underlined part. The first of these repeats the original; the other four are
different. If you think the original is best, choose the first answer; otherwise
choose one of the others.
131. George’s personal diary and album formed the basis for his book about
the places he had visited.
(a) the basis for his book about the places he had visited.
(b) the basis for his book regarding the places he had visited.
(c) the basis of his book about the places he visited.
(d) the basis of his book about the places which he had visited.
(e) the basis of his book about the places he had visited.

© 2016 SMART Training Resources Pvt. Ltd.


Directions for Q131 to Q150: In each sentence below, a part has been
underlined. Beneath the sentence you will find five ways of phrasing the
underlined part. The first of these repeats the original; the other four are
different. If you think the original is best, choose the first answer; otherwise
choose one of the others.
132. By next month Ms. Jones will be Mayor of Tallahassee for two years.
(a) will be Mayor of Tallahassee
(b) will have been Mayor of Tallahassee
(c) will be mayor of Tallahassee
(d) will have been mayor of Tallahassee
(e) could have been mayor of Tallahassee

© 2016 SMART Training Resources Pvt. Ltd.


Directions for Q131 to Q150: In each sentence below, a part has been
underlined. Beneath the sentence you will find five ways of phrasing the
underlined part. The first of these repeats the original; the other four are
different. If you think the original is best, choose the first answer; otherwise
choose one of the others.
133. If he would have revised his first draft, he would have received a better
grade.
(a) would have revised
(b) had revised
(c) could of revised
(d) had of revised
(e) would revise

© 2016 SMART Training Resources Pvt. Ltd.


Directions for Q131 to Q150: In each sentence below, a part has
been underlined. Beneath the sentence you will find five ways of
phrasing the underlined part. The first of these repeats the original; the
other four are different. If you think the original is best, choose the first
answer; otherwise choose one of the others.
134. Anu claims that cats made the best pets.
(a) made the best pets.
(b) could be the best pets.
(c) are the best pets.
(d) make of the best pets
(e) make the best pets.

© 2016 SMART Training Resources Pvt. Ltd.


Directions for Q131 to Q150: In each sentence below, a part has been
underlined. Beneath the sentence you will find five ways of phrasing the
underlined part. The first of these repeats the original; the other four are
different. If you think the original is best, choose the first answer; otherwise
choose one of the others.
135. The government requires that these forms should be submitted before
the end of the financial year.
(a) that these forms should be submitted
(b) that these forms be submitted
(c) for these forms to be submitted
(d) these forms submission
(e) these forms should be submitted

© 2016 SMART Training Resources Pvt. Ltd.


Directions for Q131 to Q150: In each sentence below, a part has
been underlined. Beneath the sentence you will find five ways of
phrasing the underlined part. The first of these repeats the original; the
other four are different. If you think the original is best, choose the first
answer; otherwise choose one of the others.
136. A conjunction is used to connect words and sentences together.
(a) words and sentences together.
(b) words or sentences together.
(c) words and sentences.
(d) words or sentences.
(e) words to sentences.

© 2016 SMART Training Resources Pvt. Ltd.


Directions for Q131 to Q150: In each sentence below, a part has been
underlined. Beneath the sentence you will find five ways of phrasing the
underlined part. The first of these repeats the original; the other four are
different. If you think the original is best, choose the first answer; otherwise
choose one of the others.
137. Citing the recent rise in violence against foreigners, the Embassy
urged expats to remain vigilant, avoid the Blackburn district, and refraining
from travelling late at night.
(a) refraining from travelling late at night
(b) travelling during the day instead of the night
(c) as an imperative, refrain from travel at night
(d) as an imperative, refrain from travel at night
(e) refrain from travelling at night

© 2016 SMART Training Resources Pvt. Ltd.


Directions for Q131 to Q150: In each sentence below, a part has
been underlined. Beneath the sentence you will find five ways of
phrasing the underlined part. The first of these repeats the original; the
other four are different. If you think the original is best, choose the first
answer; otherwise choose one of the others.
138. The protestors campaigned against whale hunting, didn’t they?
(a) didn’t they
(b) weren’t they
(c) were they
(d) isn’t it
(e) wasn’t it

© 2016 SMART Training Resources Pvt. Ltd.


Directions for Q131 to Q150: In each sentence below, a part has been
underlined. Beneath the sentence you will find five ways of phrasing the
underlined part. The first of these repeats the original; the other four are
different. If you think the original is best, choose the first answer; otherwise
choose one of the others.
139. The sports writer questioned the skill of basketball players compared
to tennis players.
(a) skill of basketball players compared to tennis players
(b) skills of basketball players compared to tennis players
(c) skill of basketball players to tennis players
(d) skill of basketball players compared to that of tennis players
(e) skill of basketball players compared to those of tennis players

© 2016 SMART Training Resources Pvt. Ltd.


Directions for Q131 to Q150: In each sentence below, a part has been
underlined. Beneath the sentence you will find five ways of phrasing the
underlined part. The first of these repeats the original; the other four are
different. If you think the original is best, choose the first answer; otherwise
choose one of the others.
140. It is a special feature of cell aggregation in the developing nervous
system that in most regions of the brain the cells not only adhere to one
another and also adopt some preferential orientation.
(a) to one another and also adopt
(b) one to the other, and also they adopt
(c) one to the other, but also adopting
(d) to one another but also adopt
(e) to each other, also adopting

© 2016 SMART Training Resources Pvt. Ltd.


Directions for Q131 to Q150: In each sentence below, a part has been
underlined. Beneath the sentence you will find five ways of phrasing the
underlined part. The first of these repeats the original; the other four are
different. If you think the original is best, choose the first answer; otherwise
choose one of the others.
141. The reason why Sravanti wrote the letter was because she could not
contact him over the phone.
(a) wrote the letter was because
(b) wrote the letter was the reason that
(c) wrote the letter was that
(d) had written the letter was because
(e) wrote the letter was since

© 2016 SMART Training Resources Pvt. Ltd.


Directions for Q131 to Q150: In each sentence below, a part has been
underlined. Beneath the sentence you will find five ways of phrasing the
underlined part. The first of these repeats the original; the other four are
different. If you think the original is best, choose the first answer; otherwise
choose one of the others.
142. After the heavy rains last week, the water in the reservoir raised
another two feet.
(a) raised another two feet
(b) risen another two feet
(c) have risen by two feet
(d) would raise another two feet
(e) rose another two feet

© 2016 SMART Training Resources Pvt. Ltd.


Directions for Q131 to Q150: In each sentence below, a part has
been underlined. Beneath the sentence you will find five ways of
phrasing the underlined part. The first of these repeats the original; the
other four are different. If you think the original is best, choose the first
answer; otherwise choose one of the others.
143. The drama enacted by the students had many scenes which
were so humorous that it was hardly possible to keeping a straight face.
(a) hardly possible to keeping
(b) hardly impossible keeping
(c) hardly possible to keep
(d) hardly impossible to keep
(e) hardly possible for keeping

© 2016 SMART Training Resources Pvt. Ltd.


Directions for Q131 to Q150: In each sentence below, a part has
been underlined. Beneath the sentence you will find five ways of
phrasing the underlined part. The first of these repeats the original; the
other four are different. If you think the original is best, choose the first
answer; otherwise choose one of the others.
144. If you had attended the seminar you could benefit a great deal.
(a) could benefit
(b) would have benefited
(c) would benefit
(d) might benefit
(e) should have benefit

© 2016 SMART Training Resources Pvt. Ltd.


Directions for Q131 to Q150: In each sentence below, a part has
been underlined. Beneath the sentence you will find five ways of
phrasing the underlined part. The first of these repeats the original; the
other four are different. If you think the original is best, choose the first
answer; otherwise choose one of the others.
145. Each group have their own ideology which it is bound to abide by.
(a) Each group have their own ideology
(b) Every group has its own ideology
(c) Each group has their own ideologies
(d) Each group has ideologies
(e) Every group have their own ideology

© 2016 SMART Training Resources Pvt. Ltd.


Directions for Q131 to Q150: In each sentence below, a part has been
underlined. Beneath the sentence you will find five ways of phrasing the
underlined part. The first of these repeats the original; the other four are
different. If you think the original is best, choose the first answer; otherwise
choose one of the others.
146. The red cross organization appealed to the people to come forward to
help the drought victims and said that each may contribute what they can.
(a) each may contribute what they can
(b) each might contribute what they can
(c) each should contribute what they could
(d) each may contribute what he can
(e) each may contribute what each one can

© 2016 SMART Training Resources Pvt. Ltd.


Directions for Q131 to Q150: In each sentence below, a part has been
underlined. Beneath the sentence you will find five ways of phrasing the
underlined part. The first of these repeats the original; the other four are
different. If you think the original is best, choose the first answer; otherwise
choose one of the others.
147. Unlike the brown sparrow, the passenger pigeon was slaughtered
indiscriminately and became extinct in 1914.
(a) was slaughtered indiscriminately
(b) did not slaughter indiscriminately
(c) had been slaughtered indiscriminately
(d) slaughtered indiscriminately
(e) has been slaughtered indiscriminately

© 2016 SMART Training Resources Pvt. Ltd.


Directions for Q131 to Q150: In each sentence below, a part has
been underlined. Beneath the sentence you will find five ways of
phrasing the underlined part. The first of these repeats the original; the
other four are different. If you think the original is best, choose the first
answer; otherwise choose one of the others.
148. Because zebras have very striped patterns, photos can be used to
identify them when captured.
(a) very striped patterns
(b) varying stripe patterns
(c) varied striped patterns
(d) various stripe pattern
(e) striped pattern in variety

© 2016 SMART Training Resources Pvt. Ltd.


Directions for Q131 to Q150: In each sentence below, a part has
been underlined. Beneath the sentence you will find five ways of
phrasing the underlined part. The first of these repeats the original; the
other four are different. If you think the original is best, choose the first
answer; otherwise choose one of the others.
149. Shyamala had not scarcely entered when Rama shouted.
(a) not scarcely entered when
(b) scarcely entered than
(c) not hardly entered when
(d) scarcely enters
(e) scarcely entered when

© 2016 SMART Training Resources Pvt. Ltd.


Directions for Q131 to Q150: In each sentence below, a part has
been underlined. Beneath the sentence you will find five ways of
phrasing the underlined part. The first of these repeats the original; the
other four are different. If you think the original is best, choose the first
answer; otherwise choose one of the others.
150. Between these two shirts which is the least costly?
(a) which is the least costly
(b) which one is the least costly
(c) which is least costly one
(d) which is less costly
(e) which one less cost

© 2016 SMART Training Resources Pvt. Ltd.


End of Session - 6
Thank You

© 2016 SMART Training Resources Pvt. Ltd.


Verbal Ability
Session – 7
Sentence Completion - I

© 2016 SMART Training Resources Pvt. Ltd.


INTRODUCTION
Sentence Completion questions are nothing but the good old ‘Fill in the
Blanks’ type of questions that you have been tackling since primary
school!
These questions essentially test your ability to use vocabulary and
identify logical consistency among the elements in a given sentence.
You need to decipher the contextual meaning of the word from the given
statement. And remember, knowing how parts of a sentence affect one
another can help you determine the correct answer.
Context clues also help us in identifying how a word is used in
the context of a paragraph or longer passage.

© 2016 SMART Training Resources Pvt. Ltd.


Exsmple:
Indeed there had been a distant rumbling for some time, although I had
paid no attention to it. Now I started listening and I thought it sounded
like the characteristic noise of jet airplanes constantly landing and taking
off.
Therefore rumbling means ____________________________.
Clue: It sounds like a jet plane constantly taking off and landing
Answer : Rumbling means a deep thundering sound
Dictionary meaning : rumbling - a loud low dull continuous noise

© 2016 SMART Training Resources Pvt. Ltd.


4 Elements that can make Sentence Completion challenging!
Vocabulary level: There may be some words that you are not familiar with.
Grammatical complexities: Some sentences can be highly complex with a
range of grammatical elements – subordinate clauses, relative clauses,
prepositional phrases, gerunds and infinitives – in convoluted ways.
Tone: Sentences reflect the writer’s attitude towards the subject matter. A
neutral tone is easy to comprehend. However, it may be harder to figure out
writing that is ironic, condescending, playful or otherwise complex in tone.
Style: Writing styles could be formal or informal, poetic or prosaic, academic
or investigative, and so on. The word choice, imagery, repetition, rhythm,
sentence structure and length could all go into making a question that much
more challenging.

© 2016 SMART Training Resources Pvt. Ltd.


Question Types
Sentence Completion questions can be of the following types:
1. Single-Blank Questions
There is a sentence with a single blank and you are expected to choose the
best word or phrase to fill the blank and complete the sentence meaningfully.
E.g.: According to the biography, Smith was a _________ employer, giving
generous bonuses on one day and ordering pay cuts the next.
(a) indifferent (b) objective (c) unpredictable
(d) ineffectual (e) unobtrusive
Answer: The phrases following the word employer give an idea about Smith’s
character. While it is true that someone who keeps changing decisions is a
difficult employer, we need to narrow down on the precise word that reflects
his arbitrary behaviour. So let’s look at each answer option:
In this case, the answer would be option (c).

© 2016 SMART Training Resources Pvt. Ltd.


2. Double-Blank Questions
In double blank questions there are two blanks instead of one! Answer
options for such questions appear in pairs as given in the example below.
So, you will have to make sure that both the words in an answer option fit
the blanks correctly.
In a way, double-blank questions are easier than single blank questions
because you can eliminate word pairs based on the inaptness of just one
word in the pair! At the minimum, if you identify the inaptness of four
words, you could have cracked the question.

© 2016 SMART Training Resources Pvt. Ltd.


E.g.: Although Ms. Harvey's disposition was generally _________, she
could become enraged when sufficiently _________.
(a) vicious…ignored (b) serene…provoked
(c) energetic … disappointed (d) meek … complimented
(e) perverse …betrayed
Answer: The use of the word "although" at the beginning of the sentence
indicates that there must be a contrast between Ms. Harvey's usual disposition
and her disposition when she becomes enraged. The second missing term
must describe how someone could become enraged. If Ms. Harvey were
sufficiently "provoked," she might conceivably become enraged, and this
would contrast with a generally "serene," or calm disposition.
In this case, the answer would be option (b).

© 2016 SMART Training Resources Pvt. Ltd.


3. Cloze Tests/ Text Completion Questions
Text Completion questions come in the form of a passage or set of
sentences, omitting crucial words from short passages and asking the test
taker to use the remaining information in the passage as a basis for
selecting words or short phrases to fill the blanks. These are a set of
sentences or passage with multiple blanks where you will have to identify
the correct words to fill the blanks. Usually, each blank has a distinct set
of answer options. Some tests give scores for every blank correctly filled,
while others give scores only when all the blanks are correctly filled.

© 2016 SMART Training Resources Pvt. Ltd.


Approach Methodology

SMART Tip

THE BLANK STRATEGY

Bring your own word to the blank.


Locate a context clue.
Assign a positive or negative value to the blank.
Never arrive at an answer without eliminating/verifying other options.
Keep level-headed.
Say the sentence in your head to ensure it makes sense.

© 2016 SMART Training Resources Pvt. Ltd.


Mini Drill on Context Clues
Directions for Q1 to Q3: What follows is a series of answered Sentence
Completion questions, with clues and answers given below. Indicate the
type of Clue (Synonym, Antonym, or Definition). With double-blank
questions write the type of clues in order of the blanks.
1. The ________ candidate met many challenges during her arduous
campaign, but she did not ________ and ultimately attained the office
she had so passionately sought.
(a) reluctant ….. concede (b) dogged ….. falter
(c) tireless ….. defame
 Clue(s): arduous, but... attained
 Answer(s): dogged ….. falter

© 2016 SMART Training Resources Pvt. Ltd.


2. Crestfallen by having done poorly in the examination, Susan began to
question her abilities. Her self-confidence was ..........
(a) appeased (b) destroyed (c) elevated
 Clue(s): crestfallen
 Answer(s): destroyed

© 2016 SMART Training Resources Pvt. Ltd.


3. Common garlic has ________ properties ; during the First World war
medics saved thousands of lives by wrapping wounds with garlic soaked
bandages.
(a) curative (b) flavouring (c) inferior
 Clue(s): ‘;’  Answer(s): curative
Because of the way the sentence is constructed, you can tell that the
missing word must be related to saving lives
* An example of how you might approach the answer choices:
 curative? Seems to start with the word “cure”, which is related to
saving lives, so maybe! Let’s take a look at the others:
 flavoring? True, but not relevant to the question.
 inferior? I know the phrase “inferior quality” and that’s never a
positive thing, whereas saving lives is good, so probably not.

© 2016 SMART Training Resources Pvt. Ltd.


Direction Indicators
Direction cues are conjunctions or punctuations that indicate the direction a sentence
is taking.
Continuation Indicator: A question-stump CONTINUES in the direction of its
CLUES if the omitted word is similar in meaning to the words in the question.
E.g.: After the laborious trek, Haridas Kumar Pal was ______: all he wanted was a
good night’s rest.
The continuation indicator here is the colon. It is followed by a definition of the omitted
word.
Answer: exhausted
Continuation Indicators  and, so, because, therefore, since, comma, colon
Amplification Indicators  not just...but (This word pair indicates that you are looking
for two words similar in meaning, where the second is more intense than the first.)

© 2016 SMART Training Resources Pvt. Ltd.


Reverse Indicator: A question stump REVERSES itself if it goes for a word that is
opposite in meaning to the CLUES.
E.g.: Though there was tremendous chaos in the hallway, there was a perfect sense
of _____ in the exam hall.
The reverse indicator here is ‘though’. So the omitted word must be an opposite of
the clue word ‘chaos’.
Answer: order
Reverse Indicators  while, but, despite, though, although, seemingly, apparently,
not, even, contrast, differing, yet

© 2016 SMART Training Resources Pvt. Ltd.


Reverse indicators can suggest a reverse from a Clue (in a single-blank question), or
a reverse between blanks (in a double-blank question or cloze test) in the
relationship between the omitted words.
Watch out for the DOUBLE NEGATIVE effect. Two reverse indicators function
together as a
continuation indicator.
E.g.: The wedding party, while ______, could not ______ for the lack of warmth:
nobody was happy.
Reverse indicators ‘while’ and ‘not’ cancel each other out and you are left with a
simple continuation.
Answers: lavish, compensate

© 2016 SMART Training Resources Pvt. Ltd.


Mini Drill on Word Values
Directions for Q1 to Q3: Read each sentence carefully, keeping your ears
and eyes peeled for context clues. Then circle Positive word, Negative word, or
Unknown value to indicate your “word value” for the blank.
1. Regretfully, team captain Marc started to _________ the young players as
soon as basketball practice ended.
Positive word Negative word Unknown value
Additional strategy, using Active Pencil: With your pencil, jot a positive
(+) or negative (–) sign next to each answer choice as shown below. If the
word value is unknown, leave the word as is.
(A) laugh + (B) study + (C) race (D) belittle – (E) praise +
Ans: This sentence lacks a context clue that keys us into the precise meaning
of the blank. However, you can assign a negative (–) value to the blank, since
the sentence begins with a negative term, regretfully.

© 2016 SMART Training Resources Pvt. Ltd.


2. To the author’s delight, the controversial book received praise and
________ from most major critics.
Positive word Negative word Unknown value
Ans: The blank is linked to praise with the conjunction and; this tells us
that the needed word is on a par with or similar to praise. Therefore, you
would assign a + to the blank.

© 2016 SMART Training Resources Pvt. Ltd.


3. Regardless of his ________ nature, Pedro decided not to extend a
helping hand to his avaricious neighbors this time.
Positive word Negative word Unknown value
Ans: A subtle but key context clue is the word not. Downsize this sentence
to something like: Regardless of his “helpful” nature, Pedro chose NOT to
help . . . . Considering this simplified form of the sentence, you see that the
blank requires a positive word, such as obliging or accommodating.

© 2016 SMART Training Resources Pvt. Ltd.


Exercise 1
Directions for Q1 to Q8: Identify and write and clues in order of the blanks
and fill up the blank(s) with the most suitable words.
1. Although the weather report had suggested rain, the day was sunny and
________
 Clue(s):
 Answer(s):

2. Everyone needs to earn a living somehow, but even the most ________
individuals, abide by certain ________ professional standards.
 Clue(s):
 Answer(s):

© 2016 SMART Training Resources Pvt. Ltd.


3. The ________ of the device ________ the scientists : they could not
understand how an ancient civilization could produce something so
intricate.
 Clue(s):
 Answer(s):

4. John has been described as ________, subject to sudden changes of


mood and feeling.
 Clue(s):
 Answer(s):

© 2016 SMART Training Resources Pvt. Ltd.


5. The ________ of even a single celled organism is surprisingly
sophisticated, containing internal structures carefully honed over millions
of years of evolution.
 Clue(s):
 Answer(s):

6. Today Wegener's theory is ________; however, he died an outsider


treated with ________ by the scientific establishment.
(a) unrivalled – reverence (b) unchallenged – disdain
(c) dismissed – contempt
 Clue(s):
 Answer(s):

© 2016 SMART Training Resources Pvt. Ltd.


7. The application of statistical analytics to sports science has resulted in
some ________ in professional athletics; the cross disciplinary approach
has been particularly useful in developing new metrics for efficiency and
effectiveness.
 Clue(s):
 Answer(s):

8. Despite the ________ evidence against him, the accused maintained


an expression of ________ throughout the trial, apparently unfazed by
strength of the prosecutor’s case.
 Clue(s):
 Answer(s):

© 2016 SMART Training Resources Pvt. Ltd.


Directions for Q9 and Q10: Read each sentence carefully, keeping
your ears and eyes peeled for context clues. Then circle Positive word,
Negative word, or Unknown value to indicate your “word value” for the
blank.

9. Though gossip has painted an unpleasant and downright _________


picture of the new member of the community, I have learned first hand
that Ms.Appleton is actually forthright and well mannered,

Positive word Negative word Unknown value

© 2016 SMART Training Resources Pvt. Ltd.


Directions for Q9 and Q10: Read each sentence carefully, keeping
your ears and eyes peeled for context clues. Then circle Positive word,
Negative word, or Unknown value to indicate your “word value” for the
blank.

10. Contrary to his pleasant and convivial nature, Jack decided not to
attend several holiday parties; he was becoming uncharacteristically
________ and ________.
Positive word Negative word Unknown value

© 2016 SMART Training Resources Pvt. Ltd.


Directions for Q11 to Q25: Fill in the blanks with the most suitable
word from the list of options given.
11. Abreast all of this attention the author is still keeping busy wielding her
pen to produce narratives that voice the thoughts, hopes and perspectives
of those often unheard of in popular ________ literary works.
(a) offbeat
(b) prudent
(c) panache
(d) mainstream

© 2016 SMART Training Resources Pvt. Ltd.


Directions for Q11 to Q25: Fill in the blanks with the most suitable
word from the list of options given.

12. During the Victorian Age, as seen in Oscar Wilde’s ‘The importance of
being Earnest’, the English aristocracy was dominant, ________ and
rich - far removed from the British middle class and poor.
(a) servile
(b) meek
(c) snobbish
(d) fastidious

© 2016 SMART Training Resources Pvt. Ltd.


13. The tone of the play is so light-hearted that the audience never doubt
that things will end happily, and it is therefore free to enjoy the comedy
without being caught up in the tension of a/an ________ outcome.
(a) mundane
(b) nascent
(c) uncertain
(d) ambient

© 2016 SMART Training Resources Pvt. Ltd.


14. Crocodiles compete ________ with each other for territory, with
________ males in particular occupying the most eligible stretches of
freshwater creeks and streams.
(a) grudgingly, mediocre
(b) fiercely, dominant
(c) majorly, subservient
(d) belligerently, docile

© 2016 SMART Training Resources Pvt. Ltd.


15. A key asset is the country’s first-rate virology laboratory affiliated with
the University Teaching Hospital. The laboratory was staffed and
equipped to quickly and reliably diagnose a case of Ebola virus disease,
which ensured that ________ measures could begin with the shortest
possible delay.
(a) catastrophic
(b) callisthenic
(c) containment
(d) dissemination

© 2016 SMART Training Resources Pvt. Ltd.


16. When EVD is suspected in a person, his or her travel and work history,
along with an exposure to wildlife, are important factors to consider with
respect to further ________ efforts.
(a) prosthetic
(b) diagnostic
(c) humane
(d) histrionic

© 2016 SMART Training Resources Pvt. Ltd.


17. Tranquebar has the sea and nothing else. There is an old fort by the sea
which offers a ________ view of the angry, frothing waters.
(a) vilifying
(b) belligerent
(c) balsamic
(d) breath-taking

© 2016 SMART Training Resources Pvt. Ltd.


18. In an age of change, Wilde's plays ________ people to think about
the artificial barriers that defined society and enabled a privileged life for
the rich at the expense of the working class.
(a) entrusted
(b) enthused
(c) encouraged
(d) entrenched

© 2016 SMART Training Resources Pvt. Ltd.


19. “According to WHO, the success of Nigeria – Africa’s most populous
nation – was ________ to ample funding, quick action and assistance
from the WHO, the U.S. Centers for Disease Control and the non-profit
Doctors Without Borders.”
(a) asynchronous
(b) attributable
(c) synchronous
(d) detrimental

© 2016 SMART Training Resources Pvt. Ltd.


20. Although it is not entirely clear how Ebola initially spreads from
animals to humans, the spread is _____ to involve direct contact with an
infected wild animal or fruit bat.
(a) enamoured
(b) paramount
(c) believed
(d) truant

© 2016 SMART Training Resources Pvt. Ltd.


21. Though most of the conflict in the play ________ from the troubles
of romance, and though the play involves a number of romantic
________, it is not truly a love story; it distances the audience from the
emotions of the characters in order to ________ fun at the torments and
afflictions that those in love ________.
(a) stems, elements, poke, suffer
(b) dissolves, chords, hit, enjoy
(c) facilitates, scenes, make, cherish
(d) starts, gaffes, make, celebrate

© 2016 SMART Training Resources Pvt. Ltd.


22. Not many people were around at that time and I loved the silence of
the place, only ________ by the tinkling of cutlery that came from the
kitchen.
(a) punctured
(b) elevated
(c) alleviated
(d) exacerbated

© 2016 SMART Training Resources Pvt. Ltd.


23. This is the kind of journalism that is not about informing the reader
but about making sure that the readers’ real and imagined petty
________ remain undisturbed.
(a) amenities
(b) joys
(c) prejudices
(d) retributions

© 2016 SMART Training Resources Pvt. Ltd.


24. My eyes cloud over with memories as I remember how my father took
his first, hesitant baby footsteps into the world of computers. The mouse
scared him. My brother’s temper scared him even more. He would
________ make notes and follow the step by step instructions written
down by his antsy children. Ah, those were the days of Reality TV outside
the television box.
(a) copiously
(b) dementedly
(c) endemically
(d) valiantly

© 2016 SMART Training Resources Pvt. Ltd.


25. Advances in aerospace, medical, and military technologies are
destined to ________ wildlife research techniques to ever-greater levels
of ________.
(a) send, chicanery
(b) delve, artistry
(c) plunge, empowerment
(d) propel, sophistication

© 2016 SMART Training Resources Pvt. Ltd.


End of Session – 7
Thank You…
Verbal Ability
Session – 8
Sentence Completion - II

© 2016 SMART Training Resources Pvt. Ltd.


Directions for Q26 to Q30: Fill in the blanks with the most suitable
word from the list of options given.
26. American wildlife biologists began to ________ Cold War-era
surveillance technologies into their practices during the second half of the
twentieth century. VHF radio tracking began in the late 1950s, where an
animal would be captured, ________, and then fitted with a collar or tag
that contained a radio transmitter.
(a) incorporate, sedated
(b) utilize, released
(c) dismantle, tracked
(d) assemble, catered

© 2016 SMART Training Resources Pvt. Ltd.


27. As a complete beginner one can expect to gain a good 10-12 kilos of
muscle, albeit along with a very noticeable amount of fat, with a year of
consistent, ________, linear progression based training.
(a) digressive
(b) diligent
(c) esoteric
(d) ethereal

© 2016 SMART Training Resources Pvt. Ltd.


28. Thanks to a lifetime of ________ habits and asthma, my entry into
running was ________ by suffering. My first run lasted an amazing
100m at the end of which I found myself sitting on the pavement
panicking and hurriedly taking puffs from my inhaler.
(a) sedentary, dominated
(b) acerbic, accelerated
(c) opulent, decelerated
(d) querulous, endured

© 2016 SMART Training Resources Pvt. Ltd.


29. I am not going to remember my first run as the one that made me feel
________. I’m going to love every remaining second of this trail. It
doesn’t matter if I have to walk it or even if I have to call for help, but I
was going to ________ every remaining moment.
(a) mighty, cherish
(b) miserable, savour
(c) significant, endure
(d) insignificant, suffer

© 2016 SMART Training Resources Pvt. Ltd.


30. Why is there an ________ desire to lose as much weight in as
little time as possible? It is this ________ that makes you vulnerable –
vulnerable to food manufacturers who ________ you into believing their
food will let you have the cake and eat it too, vulnerable to pseudo-fitness
gurus and gyms/fitness centers which promise you results that are too
good to be true and vulnerable to a side of yourself that is always
________ you with shortcuts.
(a) insatiable, desperation, scam, tempting
(b) insane, jubilance, tweak, bludgeon
(c) urbane, stupidity, attract, market
(d) enervated, guides, enthral, gawk

© 2016 SMART Training Resources Pvt. Ltd.


Directions for Q31 to Q50: Fill in the blanks with the most suitable
word from the list of options given.
31. Egg, yolk contains choline which is extremely ________ for the
body’s proper functioning, lutein which saves eyesight, contains essential
fatty acids and healthy dose of cholesterol which, again, is a substance that
is absolutely required for the body to function smoothly.
(a) critical
(b) unimportant
(c) sensible
(d) durable

© 2016 SMART Training Resources Pvt. Ltd.


Directions for Q31 to Q50: Fill in the blanks with the most suitable
word from the list of options given.
32. The emphasis here is on staying away from all foods that could
________ hurt you and eating only foods that are ________.
(a) impressively, effective
(b) potentially, benign
(c) occasionally, addictive
(d) sedentarily, cautious

© 2016 SMART Training Resources Pvt. Ltd.


33. Health is not a short-term goal. Health is the ________ result of
many years of eating good food among other things like leading an active
and stress-free lifestyle.
(a) consuming
(b) consummate
(c) cumulative
(d) catered

© 2016 SMART Training Resources Pvt. Ltd.


34. Countless falls triggering false alarms, innumerable bruises calling for
Dettol and Soframycin and scars that serve as battle wounds till today
were a part of my learn-to-run process. But then, a ________ skill was
learnt.
(a) pent
(b) priceless
(c) plentiful
(d) passable

© 2016 SMART Training Resources Pvt. Ltd.


35. Soy processing isn’t a very ________ picture with acid washing and
neutralization solutions, large and leaching aluminium tanks, and high
temperature heating. And this doesn’t take into ________ the artificial
flavours, including MSG, that are ________ added to improve flavour.
(a) comforting, account, oftentimes
(b) pretty, interaction, rarely
(c) disconcerting, attrition, incidentally
(d) pleasing, consideration, never

© 2016 SMART Training Resources Pvt. Ltd.


36. It’s vital to go organic with food today. A lot of foods are genetically
modified; they have high pesticide ________ levels.
(a) Bolster
(b) contamination
(c) accommodation
(d) derision

© 2016 SMART Training Resources Pvt. Ltd.


37. Irrespective of how ________ a product is, it is your duty as a parent
(and as a person who has common sense) to look into the list of
ingredients and make smart choices.
(a) fallible
(b) gullible
(c) convincing
(d) insensible

© 2016 SMART Training Resources Pvt. Ltd.


38. While most mushrooms sold to the public today are edible, one has got
to be very cautious about mushroom that is found naturally. It can be
________!
(a) palatable
(b) delicious
(c) dexterous
(d) poisonous

© 2016 SMART Training Resources Pvt. Ltd.


39. It is ________ that a road journey of less than six hours from hot
Chennai takes one to a very cool Bangalore.
(a) surprising
(b) credible
(c) laudable
(d) pliable

© 2016 SMART Training Resources Pvt. Ltd.


40. I loved the movie to bits; it is ________ to watch an Indian woman
who learns the importance of having fun and does not end up dead
because she dared to do so.
(a) shocking
(b) unreal
(c) liberating
(d) idyllic

© 2016 SMART Training Resources Pvt. Ltd.


41. There really is no need for a woman to stop being ambitious because
she assumes someday, when she has a family, she won’t be able to
________ her ambition along with them.
(a) alleviate
(b) accommodate
(c) deviate
(d) digress

© 2016 SMART Training Resources Pvt. Ltd.


42. The book also provides a great reading list for those who want to study
the research cited in her book more ________.
(a) extensively
(b) independently
(c) fiscally
(d) geographically

© 2016 SMART Training Resources Pvt. Ltd.


43. To my dad, getting the first rank was an ________. So much so that
my brother and I followed this policy when we were kids – there are two
entrances to our house: one through my dad’s office room where he
________ his clients and one through the drawing room of the house. If
we won the first prize, we’d make a ________ entry through the office
room so my dad could tell all his junior lawyers, clerk, typist, sundry
clients what geniuses we were. If we didn’t win, we’d ________ slip in
through the other entrance.
(a) admonition, treats, slinky, just
(b) obsession, receives, grand, quietly
(c) addiction, shuns, frantic, noisily
(d) attraction, meets, calm, maniacally

© 2016 SMART Training Resources Pvt. Ltd.


44. I remember one Mother’s Day when I’d written a poem and made a
card for my mother. She looked at it and said, ‘I don’t want all these
meaningless ________. Why don’t you clean your toilet?’
(a) gestures
(b) tricks
(c) pointers
(d) ruses

© 2016 SMART Training Resources Pvt. Ltd.


45. Chua, in her book, says Chinese parents push their children so much
because they believe their children can excel and all they are doing is to
help them ________ their potential. Western parents, on the other
hand, are ________ with letting the child choose how much potential it
wanted to realize.
(a) reclaim, stupid
(b) respond, selfish
(c) realise, content
(d) ruining, happy

© 2016 SMART Training Resources Pvt. Ltd.


46. Sibling rivalry is an issue Chua talks about in the book. For Western
parents, comparing their children is unthinkable. But Chinese parents do
it to ________ the underachiever to be more like the successful kid.
(a) gauge
(b) dissuade
(c) imitate
(d) encourage

© 2016 SMART Training Resources Pvt. Ltd.


47. Throughout my childhood, my brother was the ________ I was
supposed to ________. He was one of those annoying kids who don’t
seem to study at all but are just brilliant naturally. I used to wonder if my
mum ate a specially blessed mango when she was pregnant with him.
(a) paragon, emulate
(b) stalwart, dislike
(c) example, be
(d) wastrel, simulate

© 2016 SMART Training Resources Pvt. Ltd.


48. Children believe you are responsible for their boredom. If you don’t do
something to ________ it immediately, they are going to protest.
(a) forfeit
(b) ensue
(c) dispel
(d) progress

© 2016 SMART Training Resources Pvt. Ltd.


49. The men had a ________ dislike for the captain appointed recently.
(a) sleek
(b) particular
(c) omnipresent
(d) ominous

© 2016 SMART Training Resources Pvt. Ltd.


50. Languages change over time. Healthy, living languages change a lot
over time. They ________ and flow and morph.
(a) elk
(b) ebb
(c) eke
(d) era

© 2016 SMART Training Resources Pvt. Ltd.


End of Session - 8
Thank You

© 2016 SMART Training Resources Pvt. Ltd.


Verbal Ability
Session – 9
Cloze Test
CLOZE TEST

• A Cloze test consists of a text passage with some words removed


(cloze text). It’s a mixture of comprehension and fill in the blanks
type of questions.
• To solve cloze test correctly, you are expected to have a strong
command over the language and grammar, along with good
vocabulary.
Tricks to solve the cloze test

a) Read the passage at least twice without filling up the blanks.


b) Don’t treat each sentence separately instead think of logical
connections that link up the sentences together.
c) Fill up those blanks that you are 100% sure.
d) To find out the missing words in the remaining gaps, find out
which among the following part of speech will fill in the
gap: articles, nouns, pronouns, adverbs, prepositions, adjectives,
conjunctions or verbs.
Tricks to solve the cloze test

e) Some sentences may have the following combinations:


• a preposition following a noun, adjective or verb. (Example: good at
languages)
f) a prepositional phrase. (Example: in spite of )
g) an adverb. ( Example: He moved to Mumbai two years ago.)
h) a connector.(Example: First, he arrived; then he sat down; finally, he
left.)
i) a conjunction. (Example: Although he is seven, he can speak
eight languages.)
j) an auxiliary verb. (Example: He has won 2 contests.)
k) an article or some other kind of determiner. (Example: I have no time.)
Tricks to solve the cloze test
l) a pronoun, either subject or object. (Example : It is easier to know.)
m) a comparative or superlative involved?
n) Check Tone – The passage is usually written in a certain tone; sometimes
narrative, sometimes critical, sometimes humorous. Pick words that fit in with
the tone of the passage.
• For example –
Jonah ____ down the stairs, bumping along like a quarter in a tumbling dryer.
(a) tumbled (b) fell (c) dropped
• Clearly, you can use either ‘tumbled’ or ‘fell’ in this blank. But the rest of the
sentence is written in a humorous vein. So we try to maintain the tone of the
sentence. This is best accomplished by the use of the word ‘tumbled’ as it brings
to mind images of people falling funnily.
In the following passages there are blanks, each of which has been
numbered. These numbers are given below the passages and against
each, four or five words are suggested, one of which fits the blank
appropriately. Find out the appropriate word in each case.
The Huffington post __(1)__ its first international edition, Huffpost Canada, on May 26,
2011. On July 6 of the same year, the Huffington Post UK launched its UK __(2)__. On
January 23, 2012, Huffington, in partnership with Le Monde and Les Nouvelles Editions
Independantes, launched Le Huffington Post, and the launch of French __(3)__ edition is
the first in a non-English speaking __(4)__. On February 8, another French language edition
__(5)__ launched in the Canadian province of Quebec. On May Day, a US-based Spanish-
language edition was launched under the name Huffpost Voces, replacing AOL’s Hispanic
news platform, AOL Latino.
1. (a) launched (b) demanded (c) installed (d) took (e) established
2. (a) platform (b) process (c) edition (d) label (e) stint
3. (a) translation (b) language (c) post (d) term (e) module
4. (a) province (b) region (c) area (d) country (e) group
5. (a) was (b) is (c) were (d) has (e) had
Once upon a time, two friends were __(6)__ through the desert. During
some point of the __ (7)__ they had an argument, and one friend slapped
the other one on the face. The one who got slapped was ___ (8)__ but
without saying anything, he wrote in the sand, “Today my best friend
slapped on the face”. They kept on walking __(9)__ they found an oasis
where they __(10)__ to take a bath.
6. (a) crawling (b) speaking (c) swimming (d) walking (e) dancing
7. (a) journey (b) touring (c) travelling (d) border (e)
hunt
8. (a) dead (b) captured (c) presentable (d) missing (e) hurt
9. (a) as (b) until (c) from (d) with (e) through
10. (a) decided (b) resolved (c) promised (d) want (e) decide
The one, who had been slapped, got __(11)__ in the quicksand and started being
drawn inside. But the other friend acted fast and pulled him out. After being
__(12)__ from nearly being drowned, he wrote on a stone, “Today my best friend
saved my life”. The friend who had slapped and then saved his best friend asked him,
“After I hurt you, you wrote in the sand but __(13)__ you wrote on a stone. Why?
The other friend __(14)__ “When someone hurts us, we should write it down in
sand where winds of forgiveness can erase it away. But, when someone does
something good for us, we must __(15)__ it on a stone where no wind can ever erase
it”
11. (a) murdered (b) stuck (c) blended (d) mixed (e) marooned
12. (a) caught (b) doomed (c) died (d) recovered (e) saved
13. (a) also (b) immediately (c) soon (d) today (e) now
14. (a) called (b) tell (c) replied (d) questioned (e) asked
15. (a) publish (b) scratch (c) engrave (d) cypher (e) bury
Whenever I ___(16)__ at the moon, my heart __(17)__ with pleasure. I
wish I had ___(18)___ to fly up to the moon, I know if I could __(19)_ to
become an __(20)__, I would be able to go to the moon. Special equipment
and a __(21)___ training are required to go to the moon. I would have to
wear a special __(22)___ along with a gas-mask for ___(23)__. My space
shuttle would be equipped with ___(24)___ cameras and other scientific
___(25)___.
16. (a) looked (b) saw (c) viewed (d) gaze
17. (a) fills (b) filled (c) field (d) feels
18. (a) hands (b) wings (c) legs (d) feelers
19. (a) grow up (b) mow (c) blow (d) show
20. (a) astrology (b) astronomy (c) astronaut (d) aristocrat
Whenever I ___(16)__ at the moon, my heart __(17)__ with pleasure. I
wish I had ___(18)___ to fly up to the moon, I know if I could __(19)_ to
become an __(20)__, I would be able to go to the moon. Special equipment
and a __(21)___ training are required to go to the moon. I would have to
wear a special __(22)___ along with a gas-mask for ___(23)__. My space
shuttle would be equipped with ___(24)___ cameras and other scientific
___(25)___.
21. (a) special (b) ordinary (c) common (d) natural
22. (a) trousers (b) spacecraft (c) space-suit (d) space formula
23. (a) breadth (b) breathing (c) bathing (d) breathe
24. (a) sensitive (b) touchy (c) primitive (d) decorative
25. (a) accessory (b) things (c) intuition (d) gadgets
Given the state of __(26)___ life in our country today—where corruption,
violence and intolerance are increasingly __(27)___ in day-to-day
interactions isn’t it __(28)__ time that ___(29)__ made a concerted effort
to teach values? The very first __(30)__ from many is that values cannot be
taught, they need to be __(31)__. Ideally, values must be taught by people
who are ___(32)___ to lead by example. Fortunately for us, Indian
culture has ensured that good role ___(33)___ can still be found
___(34)___ our midst. We only need to ___(35)___ in on them.
26. (a) my (b) their (c) social (d) night (e) public
27. (a) sordid (b) practical (c) evident (d) theoretical (e) tense
28. (a) long (b) high (c) quantum (d) quality (e) depressing
29. (a) educationists (b) players (c) politicians (d) artists (e) government
30. (a) suggestion (b) objection (c) proclamation (d) temptation (e) emotion
Given the state of __(26)___ life in our country today—where corruption, violence
and intolerance are increasingly __(27)___ in day-to-day interactions isn’t it
__(28)__ time that ___(29)__ made a concerted effort to teach values? The very
first __(30)__ from many is that values cannot be taught, they need to be __(31)__.
Ideally, values must be taught by people who are ___(32)___ to lead by example.
Fortunately for us, Indian culture has ensured that good role ___(33)___ can still
be found ___(34)___ our midst. We only need to ___(35)___ in on them.
31. (a) studied (b) tried (c) learned (d) imbibed (e) experienced
32. (a) thrilling (b) willing (c) declaring (d) considering (e) trying
33. (a) players (b) actors (c) models (d) reversal (e) leading
34. (a) amongst (b) between (c) above (d) like (e) against
35. (a) ten (b) bang (c) zero (d) hit (e) smart
Economic growth figures for the first quarter of this financial year seem to
support the claim that the worst may be over for the Indian Economy. The
gradual revival is also an indication that the government’s economic stimulus
package is __(36)__. What could, however, upset the positive outlook is the
drought which __ (37)__ large parts of the country and its impact on overall
growth. Even though the monsoon had picked up ___(38)__ the rains received
were grossly __ (39)__. There are clear __(40)__ that farm output, particularly
cereals, will fall drastically. Insufficient rain is bound to shoot up the __ (41)__
of agricultural commodities and that would impact the economy as a whole.
……………….
36. (a) impractical (b) ambiguous (c) failing (d) working (e) weakening
37. (a) strike (b) affected (c) exposed (d) reverted (e) altered
38. (a) unseasonably(b) unfavorably (c) presently (d) meagerly (e) later
39. (a) inadequate (b) enough (c) missing (d) ample (e) atrocious
40. (a) contradictions (b) advices (c) reasons (d) results (e) indications
………..There are clear __(40)__ that farm output, particularly cereals, will fall
drastically. Insufficient rain is bound to shoot up the __ (41)__ of agricultural
commodities and that would impact the economy as a whole. The drought would also
__(42)_ a drastic reduction in rural employment and consumption besides inflation
in the prices of food articles.
Food prices have been _ (43)_ since the past few months, and lower agricultural
production is likely to _(44)_ the situation. The government has said that food grain
from the buffer stocks will be used to keep prices __(45)__. Subsidized food grain is
necessary in these times, but its effectiveness will depend a lot on the distribution
system.
41. (a) production (b) requirement (c) prices (d) yield (e) labour
42. (a) trigger (b) lead (c) result (d) contribute (e) dampen
43. (a) improving (b) balanced (c) stable (d) increasing (e) decreasing
44. (a) aggravate (b) amend (c) smoothen (d) improve (e) challenge
45. (a) unprofitable(b) futile (c) maximum (d) growing (e) down
End of Session – 9
Thank You…
Verbal Ability
Session – 10
Para-Jumbles- I
1. What are para jumbles?

• Para jumbles are, as the name suggests, jumbled paragraphs. The


question comprises of a paragraph – but the sentences are not in the
right order. You are expected to rearrange the sentences so that they
make logical sense.
• Normally instructions for this type of questions will read "Choose
the most logical order of sentences from among the given choices to
construct a coherent paragraph".
• Solving para jumbles involves a strong grasp of context, language,
grammar and more importantly, it requires common sense.
2. Question Types

Most para jumble questions are of two types:


1. Moving jumbles: This is the plain vanilla version where a bunch of
sentences are jumbled and given to you. And the answer is a certain
combination of these sentences.
2. Anchor jumbles: In an anchor jumble, the first and the last sentence
are given. And the sentences in between are in random order. The
answer is again, a specific combination of these sentences.
3. Approach Methodology

The different methodology adopted for solving para jumbles are:


(a) Acronym Approach - full form vs. short form
(b) Time Sequence Approach - TSA - either dates or time sequence indicating words
(c) Examples Approach - EA - after an hypothesis or theory
(d) Articles - definite and indefinite
(e) Noun, Pronoun, and Demonstrative Adjective - NPDA Approach - limited to not just noun
(f) Opening - Closing Sentence Approach - OCSA - supported or free, general or need previous
explanation
(g) Key Words Approach - KWA - words repeated in two consecutive sentences
(h) Structure Approach - SA - link sentences logically
(i) Indicating Words Approach - IWA - take care of words that indicate something helpful to
decide the sequence
4. What to see while doing para jumbles
(a) Even the toughest para jumbles have two or three sentences that form a link. Try to spot a link
that has to go together and then eliminate the options that do not have that link. This is the first
round of elimination.
(b) Conjunctions play a very important role in joining two or more sentences. Beware of all
the subordinating conjunctions such as after, although, as, as far as, as if, as long as, as soon
as, as though, because, before, if, in order that, since, so, so that, than, though, unless,
until, when, whenever, where, whereas, where ever and while.
(c) The coordinating conjunctions are also very important. Coordinating conjunctions are: for,
and, nor, but, or, yet and so.
(d) Keep an eye on adverbs like even, also, still, however, nevertheless, notwithstanding
etc. Such adverbs help link two or more sentences.
(e) Use the pronouns to your advantage. In long paragraphs the pronoun reference may not always
work. Ideally a pronoun in one sentence should always point to a noun in the sentence
immediately preceding it.
4. What to see while doing para jumbles
(f) General information always precedes specific information. That is the thumb rule.
Information/idea that is more universal should come first followed by information/idea
that is less universal or more specific.
(g) Information of an idea/entity comes first followed by general description.
Tangible descriptions should always take precedence over intangible descriptions. If you
describe a man’s personality, describe his physical features and then move to his
intangible aspects, same rule applying to other things.
(h) The conclusion as a rule, always comes at the end. Any sentence that follows a
conclusion creates a break in the flow of ideas, a feeling of this awkwardness should help
you in finding the last sentence of the paragraph.
(i) Keep an eye on the chronology and the importance of events. An event of
lesser significance comes later but an event of greater significance always comes first.
(j) Lastly never rush with your answers. Be patient. Mark the answer only after you have
confidently evaluated the other options.
5. Establishing link between two sentences and
then examining the options
Suppose you establish the link 'BA'. The given options are:
(a) DABC (b) ACDB (c) CBAD (d) DBAC.
Now you are left with option (c) and (d) to examine. You read the
sentences in the order given by these two options and use your methods
again to determine which one is correct.
Is establishing links between two sentences easy?
Directions for Q1 to Q20: Each question has a number of sentences
which, when properly sequenced, form a coherent paragraph. Each
sentence is labeled with a letter. Choose the most logical order of sentences
from among the given choices to construct a coherent paragraph.
1. A. Tigers have been known to eat up to 60 pounds (27 kilograms) of meat
in one night, but more often they consume about 12 pounds (5 kilograms)
during a meal.
B. It may take days for a tiger to finish eating its kill. The cat eats until it's
full, and then covers the carcass with leaves and dirt.
C. When it's hungry again, the tiger comes back to feed some more, until
the meat is gone.
(a) ABC (b) BAC (c) BCA (d) None of these
2. A. Reasons include demand for tiger fur as well as for other body
parts that many people use in traditional medicines.
B. Tigers' habitat has also dwindled seriously as humans have
developed land for uses such as farming and logging.
C. The whole tiger species is endangered throughout its range.
(a) ABC (b) BAC
(c) CAB (d) None of these
3. A. Connie St Louis, who wrote the original account of Sir Tim’s
speech which prompted the row that led him to resign from his posts at
University College London and the Royal Society, told The London
Evening Standard she has received “unpleasant” emails and tweets
from his supporters.
B. The journalist who accused Nobel Prize-winning scientist Sir Tim
Hunt of sexism said she has been the victim of Twitter trolls and
accused the “establishment” of “bullying” and “protecting their own”.
C. Ms St Louis, a lecturer in science journalism at City University,
added: “It doesn’t make me change my mind. I am standing by my
story.”
(a) ABC (b) CAB (c) BCA (d) BAC
4. A. Hamlet is an enigma.
B. His challenge to Guildenstern rings true for everyone who seeks to
know him: "You would pluck out the heart of my mystery."
C. None of us ever really does.
D. No matter how many ways critics examine him, no absolute truth
emerges.
(a) ABCD (b) ADBC
(c) BCAD (d) None of these
5. A. The results, presented on Tuesday at the Alzheimer’s Society annual research
conference in Manchester, have been hailed as “hugely promising” because they
involve medicines that are already known to be safe and well-tolerated in people –
potentially cutting years from the timeline for drugs to reach patients.
B. Speaking ahead of her presentation, Giovanna Mallucci, professor of clinical
neuroscience at the University of Cambridge, said: “It’s really exciting. They’re
licensed drugs.
C. This means you’d do a straightforward basic clinical trial on a small group of
patients because these are not new compounds, they’re known drugs.”
D. Two licensed drugs have been shown to halt brain degeneration in mice,
raising the prospect of a rapid acceleration in the search for a medicine to beat
Alzheimer’s disease.
(a) ACBD (b) BDAC (c) CDAB
(d) DABC
6. A. The latest research suggests that the build-up of abnormal amyloid
proteins seen in Alzheimer’s, Parkinson’s and CJD are simply a visible
symptom of neuro-degeneration, but not the root cause.
B. Instead, Prof Mallucci argues, the damage really occurs when a natural
defence mechanism in the brain responds to the accumulation of plaques
by switching off the production of all new proteins, wrecking the brain’s
ability to carry out essential repairs.
C. In mice with prion disease, a neurodegenerative disorder that scientists
use as a model for dementia in mice, both treatments were shown to restore
protein production, stop brain cells from dying off and prevent memory
loss.
D. The drugs work by inhibiting an enzyme, called Perk that activates the
defence mechanism.
(a) ABCD (b) BACD (c) DCBA (d) CABD
7. A. Now, you can get a keyring-friendly mini-pack, meaning that it
can be on hand at all times.
B. It may be derided as a modish condiment, but it is extremely tasty,
and sparks the kind of devotion that prompts fans of the bright red
bottle to buy T-shirts with it on the front and say things like: “Oh, I
have it with everything.”
C. So, I decided to spend a day having it with everything.
D. Sriracha hot sauce, which originated in eastern Thailand, is a mix of
garlic, sugar, chillis, vinegar and salt.
(a) ABCD (b) DBAC (c) CADB (d) BDAC
8. A. And dropped out after the first year.
B. Deep down I knew that what I really wanted to do was to write
stories.
C. When I was growing up in Nigeria I was expected, as every student
who did well was expected, to become a doctor.
D. But I did what I was supposed to do and I went into medical school.
(a) CBDA (b) ACBD (c) DCBA (d) BCDA
9. A. This is because at 11pm on Tuesday night Greece failed to make a
€1.6bn payment due to the International Monetary Fund, one of the
country’s creditors, thus becoming the first developed nation to go into
“arrears” with the fund.
B. The effective sovereign default has ratcheted up pressure on the ECB to
further restrict the funding available to banks, which have been swapping
Greek government bonds for funding from the Frankfurt-based central
bank.
C. Greece’s banks have been shut since 29 June, after Mr Tsipras’ surprise
decision to hold a referendum prompted the European Central Bank (ECB)
to limit the emergency lending available.
D. The ECB decided to maintain the current funding limit.
(a) CABD (b) BADC (c) DCBA (d) CBDA
10. A. "My opinion is that we need to invest for something to grow, whether
that is a person, a company or a nation like Greece," Feeney explained to
i100.co.uk.
B. His reasoning: there are 503 million people in the EU. We would only
need to chip in about €3 each (the same price as a feta and olive salad) to
raise the money.
C. With Greece's government and its European creditors hitting
loggerheads over the impending deadline for its debt repayments, one 29-
year-old from Yorkshire has come up with a novel solution.
D. Thom Feeney, who says he is fed up of "dithering" politicians, has set up
a crowd funding page on IndieGoGo where he hopes to raise €1.6bn
(£1.14bn) to help bail out the country.
(a) ABCD (b) CDBA (c) ACBD (d) DCBA
11. A. A story not only allows us to understand ourselves and how we live, it
can also bring about change.
B. Forced Entertainment’s retelling of the entire Shakespearean canon
using household objects, from salt and pepper cellars to bottles of olive oil,
is entirely serious.
C. One is simply the bare bones, but a story is something more; its
meanings depend on the storyteller and how the tale is delivered, which
parts are emphasised and which downplayed.
D. As the Mahabharata tells us at its beginning: “If you listen carefully, at
the end you’ll be someone else.”
E. Not least because it points up the vast difference between telling a plot
and telling a story.
(a) ACDBE (b) BECAD (c) DACBE (d) BEACD
12. A. But seen from abroad, the UK looks a haven of stability.
B. Consumers are spending and businesses are investing.
C. In the past, news of a deficit would have alarmed foreign holders of
sterling.
D. They would have pulled their money out of the UK, prompting a fall
in the value of the pound, which would have helped boost exports and
trim imports, leading to an improvement in the balance of payments.
E. Growth was stronger in the first quarter at 0.4% than previously
believed.
(a) CDAEB (b) ABDCE (c) CADBE (d) BADCE
13. A. At the age of twenty-one he wrote a treatise upon the binomial
theorem which has had a European vogue.
B. But the man had hereditary tendencies of the most diabolical kind.
C. Moriarty is a man of good birth and excellent education, endowed by
nature with a phenomenal mathematical faculty.
D. A criminal strain ran in his blood, which, instead of being modified,
was increased and rendered infinitely more dangerous by his
extraordinary mental powers.
E. On the strength of it, he won the mathematical chair at one of our
smaller universities, and had, to all appearances, a most brilliant career
before him.
(a) ACDEB (b) ABCED (c) CAEBD (d) CEBAD
14. A. Since then, intelligence tests have been mostly used to separate
dull children in school from average or bright children, so that special
education can be provided to the dull.
B. In other words, intelligence tests give us a norm for each age.
C. Intelligence is expressed as intelligence quotient and tests are
developed to indicate what an average child of a certain age can do-what
a 5-year-old can answer, but a 4-year-old cannot, for instance.
D. Binet developed the first set of such tests in the early 1900s to find out
which children in school needed special attention.
E. Intelligence can be measured by tests.
(a) CDABE (b) DECAB (c) EDACB (d) CBADE
15. A. If the market falls to reflect our notions of equity, government
intervention may be needed to redistribute income.
B. Beyond the question of what to produce, we are also concerned
about for whom output is to be produced.
C. In general, the market mechanism tends to answer the basic
question of for whom to produce by distributing a larger share of total
output to those with the most income.
D. Although this result may be efficient, it is not necessarily equitable.
E. Is the distribution of goods and services generated by the
marketplace “fair”?
(a) BEACD (b) DCBEA (c) CBEAD (d) DACBE
16. A. I wanted to be taken seriously.
B. Not about the teaching material, because I was well prepared and I
was teaching what I enjoyed.
C. The first time I taught a writing class in graduate school, I was
worried.
D. I really wanted to wear my shiny lip gloss and my girly skirt, but I
decided not to.
E. Instead I was worried about what to wear.
F. And I was worried that if I looked too feminine, I would not be taken
seriously.
(a) ABDCEF (b) CBFDAE (c) CBEADF (d) ABCFED
17. A. One was in The Famous Victories of Henry V, created for the RSC’s First
Encounters programme, which cleverly takes a knife to both text and plot in a
significantly pared-down version of both parts of Henry IV, plus Henry V.
B. And not a single one of them was exactly as it appears on the page.
C. The second was near the beginning of Ivo van Hove’s Kings of War, performed in
Dutch, which condenses several of Shakespeare’s plays to explore the nature of
kingship and the responsibilities of leaders.
D. The last was on Sunday night, watching some of Forced Entertainment’s table-
top Shakespeare live-streamed on the Guardian’s website.
E. Were any of these performances less Shakespearean than a full, uncut
performance of the original text in English?
F. Over the last couple of weeks I’ve seen three very different versions of the scene in
Henry IV Part II, in which the future Henry V tries on his ailing father’s crown.
G. I’d say a resounding no.
(a) ACBDEFG (b) GFEDACB (c) FBACDEG (d) DEGACBF
18. A. A bowl of hummus weighing 4,090kg.
B. The Lebanese fought back, making 10,452kg worth of hummus.
C. Its response?
D. As a Palestinian citizen of Israel, he takes his food history seriously, as do
most residents of Abu Ghosh.
E. That’s why they were so affronted when in 2009 Lebanon unilaterally
declared hummus Lebanese and made a bowl of it weighing 2,056kg to prove
it.
F. The chef’s hummus has been voted by the Jewish Journal’s food blog as the
best in Israel and his village once held the Guinness world record for the
biggest bowl of hummus.
G. The village of Abu Ghosh wasn’t having it.
(a) GBCDEAF (b) FDEGCAB (c) AFBGDCE (d) ABCDEFG
19. A. Although there are large regional variations, it is not infrequent to find a large
number of people sitting here and there and doing nothing.
B. Once in office, they receive friends and relatives who feel free to call any time
without prior appointment.
C. While working, one is struck by the slow and clumsy actions and reactions,
indifferent attitudes, procedure rather than outcome orientation, and the lack of
consideration for others.
D. Even those who are employed often come late to the office and leave early unless
they are forced to be punctual.
E. Work is not intrinsically valued in India.
F. Quite often people visit ailing friends and relatives or go out of their way to help
them in their personal matters even during office hours.
(a) ECADBF (b) EADCFB (c) EADBFC
(d) ABFCDE (e) None of these
20. A. It wasn’t reviewed by any of the national newspapers or tipped to
be a Christmas chart topper.
B. But Chimamanda Ngozi Adichie’s We Should All Be Feminists might
just be the most important book you read all year.
C. Based on a TEDx talk the Nigerian writer gave in 2012, We Should
All Be Feminists is a potent tour de force on the subject of gender
equality; one that whispers rather than shouts and confides rather than
chides.
D. There is no excuse not to read it – particularly as it is just 52 pages
long.
E. A lunch break or commute would provide ample opportunity.
F. Earlier this month, a short essay by a bestselling, prize-winning
author was published with little fanfare.
(a) FBDCAE (b) FADCBA (c) FADCBE (d) FABCDE
End of Session – 10
Thank You…
Verbal Ability
Session – 11
Para-Jumbles- II
Directions for Q1 to 13: Sentences given in each question, when properly sequenced,
form a coherent paragraph. The first and last sentences are 1 and 6, and the four in
between are labeled A, B, C and D. Choose the most logical order of these four
sentences from among the five given choices to construct a coherent paragraph from
sentences 1 to 6.
1.
1. What does the state do in a country where tax morality is very low?
A. It tries to spy upon the taxpayers.
B. It investigates income sources and spending patterns.
C. Exactly what the tax authority tries to do now even if inconsistently.
D. It could also encourage people to denounce to the tax authorities any
conspicuously prosperous neighbours why may be suspected of net paying their
taxes properly.
6. The ultimate solution would be an Orwellian System.
(a) BACD (b) DBAC (c) ABCD (d) DCBA
2.
1. The fragile Yugoslav State has uncertain future.
A. Thus there will surely be chaos and uncertainly if the people fail to
settle their differences.
B. Sharp ideological differences already exist in the country.
C. Ethnic, regional, linguistic and material disparities are profound.
D. The country will also lose the excellent reputation it enjoyed in the
international arena.
6. At worst, it will once more become vulnerable to international
conspiracy and intrigue.
(a) BCAD (b) ADCB (c) ACBD (d) DBCA
3.
1. India’s experience of industrialization is characteristic of the difficulties faced by a
newly independent developing country.
A. In 1947 India was undoubtedly as underdeveloped country with one of the lowest
per capita incomes in the world.
B. Indian industrialization was the result of a conscious deliberate policy of growth
by an indigenous political elite.
C. Today India ranks fifth in the international community of nations if measured in
terms of purchasing power.
D. Even today, however, the benefits of Indian industrialization since independence
have not reached the masses.
6. In India has been a limited success; one more example of growth without
development.
(a) CDAB (b) DCBA (c) CABD (d) BACD
4.
1. The New Economic Policy comprises the various policy measures and changes
introduced since July 1991.
A. There is a common thread running through all these measures.
B. The objective is simple – to improve the efficiency of the system.
C. The regulator mechanism involving multitude of controls has fragmented the
capacity and reduced competition even in the private sector.
D. The thrust of the new policy is towards creating a more competitive environment
as a means to improving the productivity and efficiency of the economy.
6. This is to be achieved by removing the barriers and restriction on the entry and
growth of firms.
(a) DCAB(b) ABCD (c) BDAC(d) CDBA
5.
1. Commercial energy consumption shows an increasing trend and poses a major
challenge for the future.
A. The demand for petroleum during 1996-97 and 2006-07 is anticipated to 81
million tons and 125 million tons respectively.
B. According to the projections of the 14th power Survey Committee Report, the
electricity generation requirements from utilities will be about 415 billion units by
1996-97 and 824 billion units by 2006-07.
C. The production of coal should reach 303 million tons by 1996-97 to achieve plan
targets and 460 million tons by 2006-07.
D. The demand for petroleum products has already outstripped indigenous
production.
6. Electricity is going to play a major role in the development of infrastructural
facilities.
(a) DACB (b) CADB (c) BADC (d) ABCD
6.
1. Count Rumford is perhaps best known for his observations on the nature of heat.
A. He undertook several experiments in order to test the theories of the origin of
frictional heat.
B. According to the colorists, the heat was produced by the ‘’ caloric’’ squeezed out of
the chips in the process of separating them from the larger pieces of metal.
C. Lavoisier had introduced the term ‘’caloric’’ for the weightless substance heat,
and had included it among the chemical elements along with carbon, nitrogen and
oxygen.
D. In the ammunitions factory in Munich, Rumford noticed that a considerable
degree of heat developed in a brass gun while it was being bored.
6. Rumford could not believe that the amount of heat generated could have come
from the small amount of dust created.
(a) ABCD (b) CBDA (c) ACDB (d) CDAB
7.
1. The death of cinema has been predicted annually.
A. It hasn’t happened.
B. It was said that the television would kill it off-and indeed audiences
plummeted, reaching a low in 1984.
C. Film has enjoyed a renaissance, and audiences are not roughly
double of what they were a decade ago.
D. Then the home computer became the projected nemesis followed by
satellite television.
6. Why? probably because even in the most atomized of societies, we
human beings feel the need to share out fantasies and our excitement.
(a) CADB (b) BDAC (c) ABDC (d) DABC
8.
1. Visual recognition involves storing and retrieving of memories.
A. Psychologists of the Gastalt school maintain that objects are recognized
as a whole in a parallel procedure.
B. Neural activity, triggered by the eye, forms an image in the brain’s
memory system that constitutes an internal representation on the viewed
object.
C. Controversy surrounds the question of whether recognition is a single
one-step procedure or a serial step-by-step one.
D. When an object is encountered again, it is matched with its internal
recognition and thereby recognized.
6. The internal representation is matched with the retinal image in single
operation.
(a) DBAC (b) DCAB (c) BDCA (d) CABD
9.
1. The history of mammals dates back at least to Triassic time.
A. Miocene and Pliocene time was marked by culmination of several groups
and continued approach towards modern characters.
B. Development was retarded, however until the sudden acceleration of
evolutional change that occurred in the oldest Paleocene.
C. In the Oligocene Epoch, there was further improvement, with
appearance of some new lines and extinction of theories.
D. This led in Eocene time to increase in average size, larger mental
capacity, and special adaptations for different modes of life.
6. The peak of the career of mammals in variety and average large size was
attained in this epoch.
(a) BDCA (b) ACDB (c) BCDA (d) ACBD
10.
1. The wind had savage allies.
A. If it had not been for my closely fitted helmet, the explosions might
have shattered my eardrums.
B. The first clap of thunder came as a deafening explosion that literally
shook my teeth.
C. I did not hear the thunder I actually felt it – an almost unbearable
physical experience.
D. I saw lightning all around me in every shape imaginable.
6. It was raining so torrentially that I thought I would drown in mid air.
(a) BCAD (b) CADB (c) CBDA (d) ACDB
11.
1. Managers need to differentiate among those who commit an error once, those
who are repetitively errant but can be corrected, and those who are basically wicked.
A. The persons in this category will resort to sweet-talk and make all sorts of
promises on being caught, but, at the first opportunity will revert to their bad ways.
B. Managers must take ruthless action against the basically wicked and ensure their
separation from the organization at the earliest.
C. The first category needs to be corrected softly and duly counseled; the second
category should be dealt with firmly and duly counseled till they realize the danger
of persisting with their errant behavior.
D. It is the last category of whom the managers must be most wary.
6. The punishment must be fair and based on the philosophy of giving all the
possible opportunities and help prior to taking ruthless action.
(a) ADCB (b) CDAB (c) CADB (d) BDAC
12.
1. Many space enthusiasts now warn that only private enterprise will
truly drive human expansion into space, and yet America’s government
keeps ignoring them.
A. It may seem surprising, but there are large numbers of people who
would spend hundreds of thousands of dollars on a trip into space.
B. Tourism and entertainment are both possibilities.
C. Two people, so far, have spent $20 m, and another two are on their
way.
D. What might cause market forces to take up the mission?
6. Film and television companies would also spend tens of millions if
they could.
(a) DCAB (b) BACD (c) DBAC (d) ACBD
13.
1. Jane Austen died and came back as a fantasy writer.
A. The book itself has been called, by a media ever eager to summarise even 800-
pages hardcover tomes into a snappy catchphrase. “Harry Potter for adults”.
B. Unlike her previous avatar, the 21st century Ms Clarke (nee Austen) seems to be
enjoying the attention showered upon her and far from publishing her first book
under a pseudonym, has been a central performer at her own media circus.
C. The book, which she now calls her “debut”, began attracting media attention long
before publication and on release it’s been universally lauded.
D. In her new avatar, she calls herself Susanna Clarke, lives in Cambridge, and has
authored a fat historical fantasy novel set in the year 1860.
6. It’s also been praised by perhaps the best living author of British fantasy novels,
Neil Gaiman, as “unquestionably the finest English novel of the fantastic written in
the past seventy years.”
(a) BADC (b) DBCA (c) DCAB (d)
DCBA
Directions for Q14 to Q20: The first and the last parts of the sentence
are marked 1 and 6. The rest of the sentence is split into four parts and
marked-P, Q, R and S. These four parts are not given in their proper
order. Read the sentences and find out which of the five combinations
is correct.
14. 1. It depends upon the
P. a new heaven on earth or to destroy
Q. user, whether science will be used to create
R. the word in a
S. outlook and mentality of the
6. common conflagration.
(a) PQRS (b) SPQR (c) QPRS (d) SQPR
15. 1. The English
P. plundered the country
Q. and raw materials,
R. as a result, the Company became rich
S. of precious mineral wealth
6. and the Indian people became poor.
(a) PQSR (b) SQPR (c) SRQP (d) PSQR
16. 1. The power and pride
P. for in the courage
Q. of Sparta was above all
R. discipline and skill of these troops
S. in its army,
6. it found its security and its ideal.
(a) SPRQ (b) PQRS (c) QSPR (d) PRQS
17. 1. It is the
P. the careful observer
Q. apparently trivial phenomena
R. which gives even the
S. intelligent eye of
6. their value.
(a) SPRQ (b) PQRS (c) QSPR (d) PRQS
18. 1. Nuclear test explosions
P. food as well as
Q. present and future generations
R. directly injuring the
S. take place, contaminating air and water and
6. of mankind.
(a) RQSP (b) PSQR (c) SPRQ (d) PQRS
19. 1. The club is an
P. nearby municipal school, for the children
Q. proper and healthy functioning of a
R. are constantly disturbed by its
S. intolerable nuisance to the
6. bizarre activities.
(a) QSRP (b) SQPR (c) PQRS (d) RPSQ
20. 1. What the country needs
P. and change tactics
Q. who would encourage players
R. are coaches and officials
S. to read the game as it progresses
6. accordingly.
(a) RQPS (b) SPRQ (c) QSPR (d) RQSP
End of Session – 11
Thank You…

Você também pode gostar